64a7d2e328ccf000185958f0 ## 04 Mock Test Law Clate Archievers Batch 2024 Rakesh Sir Neha Kale

You might also like

Download as pdf or txt
Download as pdf or txt
You are on page 1of 59

BATCH - CLAT ACHIEVERS 2024

Mock Test [CLAT 2024] #04


[with Hints and Solutions]
DATE:1/07/2023

Duration: 120 Minutes Candidate Name: _______________


Max. Marks: 120 Admit Card No. : _______________

INSTRUCTIONS TO CANDIDATES

1. The question booklet (QB) contains 120 multiple-choice questions.


2. Write your Admit Card No. on the first page of the QB before starting the test.
3. Use the separate carbonized Optical Mark Reader (OMR) Response Sheet to
answer all the questions. Make sure to read the detailed instructions provided
with the OMR Response Sheet before starting the test.
4. If you notice any discrepancy in the QB, request the invigilator to replace both
the QB and OMR Response Sheet. Do not use the previous OMR Response
Sheet with the new QB.
5. During the test, write the QB No., OMR Response Sheet No., and sign in the
designated space/column on the Attendance Sheet provided.
6. Keep your Admit Card signed by the Invigilator, as it will be required during
the admissions process.
7. The QB for the Undergraduate Five-Year Integrated Programme is worth 120
marks. Each correct answer earns 1 mark, and each wrong answer results in
a deduction of 0.25 marks. There are no deductions for unanswered
questions.
8. You can keep the QB and the candidate's copy of the OMR Response Sheet
after the test.
The use of any unfair means, including the possession of electronic devices
such as mobile phones, headphones, and digital watches, is strictly prohibited
during the test. Impersonation or any other fraudulent practice may be a
criminal offense and will lead to disqualification and possible legal action.

[ 1]
CONTENTS OF QUESTION PAPER

Subject Q. Nos. Page No.


English Language 1 – 25 3–7
Logical Reasoning 1 - 25 8 – 14
Current Affairs Including General Knowledge 1 - 30 15 –20
Legal Reasoning 1 - 30 21– 28
Quantitative Techniques 1 - 10 29 - 30
Answer Key 1-120 31
Hints & Solution 1-120 32- 59

[ 2]
English Language
PASSAGE I 1. According to the passage, what is the primary purpose
Legal language, also known as legalese, is often of using legalese in legal communication and
criticized for being convoluted and difficult to documentation?
understand. Legalese employs a specialized (A) To create confusion and uncertainty among non-
vocabulary, Latin expressions, and formal diction that lawyers.
can present significant challenges for individuals (B) To undermine the legitimacy of the legal system.
outside of the legal profession. However, the use of (C) To ensure the precision and clarity of legal
such language is crucial to ensure the precision and concepts and documents.
clarity of legal communication and documentation. (D) To promote access to justice for individuals who
Legal language is used to make legal distinctions and lack legal expertise.
define legal concepts with absolute precision. In a
contractual agreement, for example, the use of exact 2. Which of the following statements best describes the
terminology and unambiguous language is crucial in impact of legalese on individuals outside of the legal
defining the rights and obligations of the parties profession?
(A) Legalese can make legal communication and
involved. The purpose of legalese is to minimize
documents inaccessible to individuals who lack
confusion and misunderstandings regarding the legal expertise, training, or fluency in the English
precise meaning of legal documents and the concepts language.
they convey. (B) Legalese can promote access to justice for
However, the use of legalese can also have negative individuals who have access to legal expertise.
consequences. The complex terminology and formal (C) Legalese can ensure that legal documents are
diction used in legal language can make legal understood equally by all parties involved.
(D) Legalese can minimize confusion and
communication and documents inaccessible to
misunderstandings among non-lawyers regarding
individuals who lack legal expertise, training, or legal concepts.
fluency in the English language. This can create an
uneven playing field where individuals who have 3. Which of the following statements about legal
access to legal expertise are at an advantage over language is NOT supported by the passage?
those who do not. Moreover, the use of legalese can (A) Legalese is often criticized for being convoluted
lead to confusion and uncertainty among non-lawyers, and difficult to understand
(B) Legal language employs a specialized
which can ultimately undermine the legitimacy of the
vocabulary, Latin expressions, and formal
legal system. diction.
To address these challenges, efforts have been made (C) The use of legalese can create an uneven playing
to simplify legal language and make it more field in legal proceedings.
accessible to non-lawyers. The Plain English (D) The use of legalese is necessary to make legal
movement, for instance, advocates for the use of clear, communication and documentation accessible to
non-lawyers.
simple, and concise language in legal documents and
communication. Legal professionals are also 4. Based on the passage, which of the following words is
recognizing the importance of communicating legal closest in meaning to "convoluted"?
concepts and documents in a manner that is (A) Straightforward (B) Complicated
understandable to the general public. (C) Simple (D) Easy
In conclusion, while legalese is an integral aspect of
legal language, it can present challenges for 5. What is the central idea of the passage?
(A) Legal language is complex and inaccessible to
individuals outside of the legal field. While precision
non-lawyers, which can create an uneven playing
and clarity are essential in legal communication and field and undermine the legitimacy of the legal
documentation, it is crucial to ensure that legal system.
language does not become an impediment to access to (B) The use of precise and unambiguous legal
justice. By simplifying legal language and making it language is crucial to avoid confusion and
more accessible, we can promote greater misunderstandings in legal communication and
understanding and transparency within the legal documentation.
(C) Legal language is a tool for making legal
system, and thus, increase access to justice.
distinctions and defining legal concepts with
absolute precision, but it can also have negative
consequences.
[ 3]
(D) Efforts have been made to simplify legal 6. What is the author's main argument in the passage?
language and make it more accessible to non- (A) Loneliness is a significant health risk, but
lawyers in order to increase access to justice. governments may not take adequate measures to
address it.
PASSAGE II (B) Loneliness is a universal problem that affects rich
Way back in 1964, when the then US surgeon general and poor countries equally.
linked cigarettes to cancer and heart disease, his report (C) Healthcare deficits are the reason that countries
went on to impact healthcare policies all around the like India lack sufficient data on loneliness.
world. Today‘s surgeon general Vivek Murthy‘s (D) Solitude should be a choice for everyone, and
advisory is that loneliness is as dangerous as smoking, intergenerational family cohesion may not be
and significantly raises the risks of heart disease as desirable for some
well as stroke. But it‘s likely to move only a few rich
countries to shift a little expenditure towards social 7. What is the reason that measuring loneliness is
connection targets. This is not at all because ―the considered imprecise?
epidemic of loneliness and isolation‖ is frozen at the (A) The interview location and interviewer's
rich west‘s boundaries. Neither poverty nor geography characteristics can affect responses.
provide any immunity at all. (B) Loneliness is a subjective experience that cannot
Still, measuring loneliness is widely seen as inherently be quantified.
imprecise. Variations such as the interview‘s location (C) Variations in healthcare policies can affect
and the interviewer‘s characteristics can majorly loneliness levels.
swing responses, which in any case can reveal more (D) Poverty and geography provide immunity to
about normative desires than what the respondent loneliness.
feels. Countries like India anyway lack the kind of
deep associative data that Murthy can cite, which is 8. According to the passage, why might Vivek Murthy's
reflective of a larger healthcare deficit here. In the recommendations for addressing loneliness face
most populous state of UP, out-of-pocket estimates challenges in the US?
are an eye-watering 71% of total health expenditure. (A) Polarisation has unhealthily shrunk discussion
In the US, on the other hand, even if monies were networks in the US.
allocated for schemes Murthy recommends, what he (B) US healthcare policies do not allocate sufficient
doesn‘t address is how these would sidestep the funding for social connection schemes..
―polarisation‖ that has unhealthily shrunk discussion (C) Solitude as a choice is not desirable in the US.
networks in the first place. (D) The US lacks sufficient data on loneliness.
Finally, and most importantly in the Indian context,
solitude as a choice should not get extinguished before 9. Which of the following statements best summarizes
it has even lived. Remember how the forced domestic the author's opinion on loneliness?
confinement of Covid was nightmarish for many, even (A) Loneliness is a universal problem that needs to be
if many others felt acutely thankful for how family addressed urgently, but there are no effective
held them through the pandemic.When the U.S. solutions currently available.
surgeon general at the time made the connection (B) Loneliness is a serious issue, but it is difficult to
between smoking and cancer and he art disease in measure and addressing it may require re-
1964, his findings had a huge impact on global health evaluating our values and priorities.
care policies and increased the risks of both heart (C) Loneliness is not as dangerous as smoking, and it
disease and stroke. When the court orders a feared is largely a personal choice that should be
criminal to be kept in solitary confinement, it is meant respected.
for them to face acute loneliness without payroll, (D) Loneliness is a problem that is confined to certain
which is amount to capital punishment against the geographic regions and socio-economic groups,
brutal forces. Even the feared criminal would have and it can only be addressed through community-
fear of falling prey to Law enforcement agencies to be led initiatives.
transferred to a prison cell or loneliness.
Consequently, Surgeon General Vivek Murthy's latest 10. What is the tone of the passage?
discovery has those in positions of authority must take (A) Cynical
precautions against healthy risks to the entity and (B) Optimistic
society because loneliness and isolation are equally as (C) Indifferent
harmful as cigarettes. This argument may be in its (D) Cautionary
early stages, but more research is required to
determine its effects before coming to a logical PASSAGE III
conclusion. Aaron Rodgers chatter has overtaken the NFL, the
most popular sports enterprise in America, in recent
years. Whether it was his MVP-play or eye-popping
contract extension or feud with his coach in Green
Bay or COVID vaccine skepticism or use of
psychedelics or his entry into a darkness retreat this
[ 4]
offseason, Rodgers, 39, has given more than enough (C) They had a successful season overall.
grist for every sports radio and TV shoutfest across (D) They had a difficult season due to injuries.
the land. Now, the future Hall of Fame quarterback
and all-time NFL lighting rod is headed to New York 14. What can be inferred about Allen Lazard?
City, the media capital of the United States. The (A) He played for the Jets before joining the Green
Green Bay Packers and New York Jets have agreed to Bay Packers.
a deal that sends Rodgers along with Green Bay‘s first (B) He is the most talented wide receiver in the NFL.
round pick (No. 13 overall) and fifth round pick (No. (C) He is a free agent who has not yet signed with a
170) in the 2023 draft to the Jets in exchange for their team.
2023 first round pick (No. 15 overall), a 2023 second- (D) He has played with Aaron Rodgers before.
round pick (No. 42), a 2023 sixth-round pick (No.
207) and a conditional 2024 second-round pick that 15. Which sentence correctly demonstrates subject-verb
becomes a first-round pick if Rodgers plays at least agreement in the passage?
65% of New York‘s offensive snaps this season.
(A) Aaron Rodgers and the Jets was making
Rodgers arrives in the Big Apple just in time to steal
New Yorkers‘ attention away from this week‘s NFL headlines.
Draft, the New York Knicks‘ strong playoff (B) The Jets, along with their draft picks, was
performance, the New York Rangers-New Jersey exchanged for Rodgers and Green Bay‘s draft
Devils NHL playoff series, the sudden firings of two picks.
prominent cable news anchors, the city‘s rat issues, (C) Rodgers, as well as his teammates, was excited
and on and on and on. Rodgers has an aptitude for about the trade.
stirring pots. Many Jets fans, however, are just plain
(D) Green Bay and the Jets have agreed to a deal that
giddy that he‘ll be wearing green—without Packer
gold—this upcoming season. And who can blame send Rodgers to New York.
them? The team‘s latest in a long line of would-be
franchise quarterback saviors—Zach Wilson, the No. PASSAGE IV
2 overall selection in the 2021 NFL Draft—was The textbook revisions that have been recently
benched last season, due to poor play. The Jets undertaken by the National Council of Educational
finished 7-10 in 2022, but have some strong pieces. Research and Training (NCERT) have evoked mixed
The team‘s defense was ranked No. 4 in the NFL a
reactions among educationists and commentators
season ago. Jets wide receiver Garrett Wilson was the
NFL‘s offensive rookie of the year. Another rookie, alike. For example, the editorial comment by Krishna
running back Breece Hall, was on the verge of a Kumar in the current issue of EPW has raised many
breakout campaign before an ACL injury ended his important issues that suggest an inadequacy of the
season. Rodgers will reunite with his top target in revisions in a vital sense. The comment rightly argues
Green Bay from a year ago, Allen Lazard, who signed that the claim to rationalise the syllabus and
a free agent deal with the Jets last month.
curriculum for reducing the burden of the
11. What can be inferred about Aaron Rodgers from the schoolchildren is only going to intensify another kind
passage? of burden on them. Taking a cue from the editorial
(A) He has never won the MVP award in the NFL. comment, one could further argue that the deletion of
(B) He has had a conflict with his coach in Green certain topics from the syllabi would make both
Bay. students and teachers hamstrung. The comment makes
(C) He has never been traded to another team before a reference to conceptual incomprehension as being a
(D) He has never shown any interest in alternative
cognitive burden; however, it is not the fault of these
medicine or therapy.
concepts but of those who have made it their business
12. Why are many Jets fans excited about Aaron Rodgers to transact with them. In other words, the knowledge
joining their team? in textbooks that is now sought to be ―rationalised‖
(A) Because he is known for being a quiet and not only leads to a conceptual incomprehension but
reserved player. also creates a moral–social burden for those who
(B) Because he will bring a lot of attention to the refuse to confront its source emanating from, say,
team.
their lack of knowledge about caste, untouchability,
(C) Because he has a reputation for being a bad
teammate. and communalism. It is an irony that the NCERT
(D) Because he has a lot of experience playing in revisions would like to solve the problem by removing
New York City. such concepts and chapters that are perceived to be
burdensome on the students, intellectually as well as
13. What can be inferred about the Jets' performance in socially. The collective consequence of such a
the previous NFL season?
conceptual incomprehension, as has been pointed out
(A) They had a losing record.
(B) They had the best defense in the NFL. in the comment, and the decision to drive out some
[ 5]
concepts and chapters from the classroom altogether, 18. According to the passage, what is the collective
tends to produce the same result in the name of consequence of conceptual incomprehension and the
rationalisation. The deletion of certain topics decision to remove certain concepts and chapters from
pertaining to the freedom struggle, social movements the classroom altogether?
against caste and untouchability, etc, would make it (A) A non-compelling knowledge base for students
correspondingly non-compelling for the framers of the and teachers.
revisions of textbooks to take the concepts of justice (B) An intensification of cognitive burden on
and equality on board for classroom discussions. students.
Deletions would make it unnecessary for the teachers (C) The production of the same result in the name of
to communicate these concepts‘ social significance rationalization.
through classroom discussions. Thus, the deletion of (D) The creation of a moral-social burden on students
such knowledge from the syllabi has far more serious and teachers.
consequences for not just the concepts and chapters
being deleted but, more importantly, for the project of 19. What is the main argument presented about the recent
shaping the normative character of our society. The textbook revisions by NCERT?
deletion of topics would lead to the elimination of the (A) The revisions have successfully reduced the
discussion and/or persuasion through deliberation of burden on schoolchildren.
normative concepts such as justice, equality, and (B) The revisions have resulted in a conceptual
secularism, which are all core constitutional values. incomprehension and a moral-social burden on
The acts of deletion render these concepts devoid of students.
textual as well as institutional context that are so (C) The revisions have made the classroom
necessary for making students aware about the discussions more compelling and effective..
infirmities of society. (D) The revisions have made the normative concepts
like justice and equality redundant..
16. What is the consequence of the NCERT's decision to
remove certain concepts and chapters from the syllabi, 20. What does the word "hamstrung" mean in the
as per the passage? passage?
(A) It intensifies cognitive burden on students. (A) Confused (B) Impeded
(B) It makes it non-compelling for the framers of the (C) Strengthened (D) Encouraged
revisions of textbooks to take the concepts of
justice and equality on board for classroom PASSAGE V
discussions. On 12 April 2023, the University Grants Commission
(C) It leads to the elimination of discussion and/or published its draft guidelines for incorporating Indian
persuasion through deliberation of normative knowledge in higher education curricula. These
concepts such as justice, equality, and secularism. guidelines, drafted in line with the National Education
(D) It creates a moral-social burden on those who Policy (NEP) 2020 mandate to promote research and
refuse to confront the source of conceptual instruction in Indian Knowledge Systems (IKS), at all
incomprehension.. levels of education, by preparing and teaching new
courses/programmes at the undergraduate and
17. What is the main concern of the passage regarding the postgraduate levels. Students both at the
NCERT textbook revisions? undergraduate and postgraduate programmes are to
(A) The cognitive burden on students due to gather at least 5% of their mandated credits from IKS
conceptual incomprehension. courses. At least half of these credits are to be sourced
(B) The deletion of topics pertaining to the freedom from IKS courses related to the student‘s major
struggle and social movements against caste and discipline of specialisation. Students of medicine, for
untouchability. instance, are expected to read a foundation course in
(C) The inadequacy of the revisions in rationalizing Indian systems of medicine followed by another
the syllabus and curriculum. elective course on a similar theme in the subsequent
(D) The elimination of normative concepts such as semester. The model curricula appended in the
justice, equality, and secularism from classroom guidelines, therefore enlist IKS foundational courses,
discussions. presumably compulsory for all students, delineating
contributions of early Indians in disciplines ranging
from architecture, metallurgy, health sciences to fine
arts and economy. What runs like a red thread through
[ 6]
the guidelines is the emphasis on an uninterrupted (D) To eliminate IKS courses from the higher
knowledge tradition right into the 19th century (see, education curriculum
for instance, the injunction ―Highlight the continuity
of the Indian astronomical tradition [1400 BCE–19th 22. According to the passage, what is the implied reason
century CE],‖ p 22), the rupture sourced ostensibly to behind the guidelines' emphasis on highlighting the
the introduction of Western education. The guidelines continuity of Indian knowledge systems right into the
picture an amalgamation of fundamental IKS concepts 19th century?
into modern textbooks as well as classroom-based (A) To create a sense of pride in India's ancient
teaching–learning activities. Indeed, the release was knowledge systems
bookended by guidelines for training/orientation of (B) To encourage students to study IKS courses and
discourage Western knowledge systems
faculty in IKS (first opened for comments in
(C) To show the long-standing history and
December 2022) uploaded on 13 April 2023 to contributions of Indian knowledge systems
―facilitate a seamless integration of Indian traditional (D) To prove that Indian knowledge systems are
knowledge with modern subjects.‖ However, one is superior to Western knowledge systems
unsure of the relationship envisaged with existing
―modern‖ curricula since subjects like Indian music, 23. Which of the following best describes the reason
Indian philosophy or even ayurveda, traditionally seen behind the UGC's draft guidelines for incorporating
Indian knowledge in higher education curricula?
as part of the IKS, are denounced for teaching
(A) To replace existing modern curricula with
―Western epistemologies and methodologies‖ (p 8); traditional Indian knowledge systems
seemingly tainted by non-Indian perspectives and in (B) To promote research and instruction in Indian
need of an overhaul. This, despite, by the now well- knowledge systems at all levels of education
known understanding, that Western knowledge has (C) To encourage students to take elective courses
long been permeated by non-Western knowledge related to Indian systems of medicine
streams, and these cultural exchanges stretch back to (D) To highlight the contributions of Western
education to Indian knowledge systems
the period much before the West achieved relative
dominance. Interestingly, the guidelines acknowledge 24. What is the tone of the passage?
and commend the spread (―outflow,‖ p 14) of science, (A) Neutral (B) Celebratory
technology, medical pharmacopoeia, and literature (C) Cynical (D) Condescending
from ancient India to Asia and Europe from ancient to
modern times, although the reverse/inflow remains 25. What is the central idea of the passage?
(A) The University Grants Commission has drafted
unacknowledged.
guidelines to promote the integration of Indian
knowledge systems into higher education
21. What is the National Education Policy (NEP) 2020 curricula.
mandate? (B) Indian knowledge systems are superior to
(A) To promote research and instruction in Indian Western knowledge systems.
Knowledge Systems (IKS) at all levels of (C) The National Education Policy (NEP) 2020 aims
education to replace Western knowledge systems with
Indian knowledge systems.
(B) To reduce the number of IKS courses in higher
(D) The introduction of Western education in India
education caused a rupture in the continuity of Indian
(C) To focus on Western knowledge systems in knowledge systems.
higher education

[ 7]
Logical Reasoning
PASSAGE I which are often felt to be in tension with one another
Over the past few decades, fewer and fewer ballots – are all on display at election time. The confluence
have been cast at precinct polling places on Election of democratic values helps citizens to appreciate
Day, more and more via mail, early voting, or other their role in a democracy.
methods of convenience voting. The US is
something of an outlier, but other democracies have 1. Which of the following statements accurately
also seen increased interest in postal voting, and it summarizes the argument made in the passage?
has been common in Switzerland for decades. (A) The shift towards alternative voting methods in
To the extent that there has been public debate about democracies has undermined the democratic
this shift, it has largely centred on questions related values of individual dignity, political equality,
to security and accessibility. While these qualities and large-scale collective action.
are essential, they do not exhaust the considerations (B) The practice of popular voting, characterized by
we should use to assess election administration. formal equality, widespread participation, and
Elections are not merely information-gathering momentous occasions, effectively highlights
exercises. They also shape citizens‘ attitudes toward and promotes democratic values.
democracy and their role within it. And popular (C) The increasing use of postal voting in
elections structure the political environment in democracies has led to a decline in citizens'
which interest groups and social movements appreciation of their role in a democracy.
organise and mobilise for collective action. For (D) The rituals of voting, such as congregating at a
these functions, the experience of voting and the local polling place, are no longer necessary in
optics of elections matter a great deal. modern democracies as they disrupt citizens'
I argue that three key features define the democratic normal routines.
practice of popular voting. First, vote-counting
procedures exhibit formal equality: they weigh votes 2. Based on the information provided in the passage,
equally regardless of who casts them. Second, which of the following assumptions can be made?
prominent elections garner uniquely widespread (A) The shift towards alternative voting methods
participation. For example, One of the United has resulted in a decline in the number of
Kingdom‘s biggest television events of recent years, citizens participating in elections.
Queen Elizabeth II‘s funeral, garnered around 30 (B) The practice of popular voting is universally
million UK viewers. Meanwhile, approximately 32 embraced by all democracies around the world.
million people voted in the 2019 UK general (C) The confluence of democratic values during
election. In today‘s democracies, people expect the election time increases citizens' trust in the
majority of their fellow citizens to vote, and wonder democratic process.
what has gone wrong when they don‘t. (D) The use of postal voting is the most secure and
Third and finally, occasions for voting are accessible method of voting in modern
momentous. I mean this in two senses: they are democracies.
time-bound, occurring in a defined political
moment; and they are attention-grabbing, disrupting 3. Which of the following statements, if true, would
our normal routines. At the same time, though, these most strengthen the argument that the practice of
momentous occasions recur regularly. They create a popular voting effectively highlights and promotes
rhythm to political life in which the ordinary, democratic values, as described in the passage?
delegated and diffuse business of politics is (A) Research studies have shown that citizens who
punctuated by extraordinary moments of mass personally participate in the act of casting their
participation. Rituals of voting, such as congregating votes develop a stronger sense of civic duty and
at a local polling place, reinforce the engagement with democratic processes,
momentousness of the occasion for those who supporting the confluence of democratic values
participate in and witness it. during election time.
Together these features constitute a practice that (B) Countries that have implemented strict voter
manages to make multiple democratic values salient identification requirements have seen an
at the same time. The values of individual dignity, increase in voter turnout and public trust in the
political equality, and large-scale collective action – electoral system, reinforcing the formal

[ 8]
equality and widespread participation aspects of (C) In countries where voting is mandatory,
popular voting. citizens' attitudes toward democracy and their
(C) Public opinion surveys consistently indicate role within it are influenced more by the
that citizens perceive election day as a obligation to vote rather than the experience of
significant event that holds symbolic value and voting or the optics of elections.
reinforces their belief in the democratic system, (D) Research has shown that citizens' attitudes
reinforcing the momentousness and large-scale toward democracy and their role within it are
collective action aspects of popular voting. primarily shaped by their socio-economic status
(D) Democracies that prioritize convenience voting and education level, with voting experiences
methods, such as mail-in ballots, have having a minimal impact on their perceptions.
experienced a decline in political apathy among
their citizens, strengthening the citizens' PASSAGE II
appreciation of their role in a democracy and The arrest of former Prime Minister Imran Khan at
the importance of widespread participation. the Islamabad High Court on May 9 triggered
massive protests across Pakistan, leading to
4. Which of the following most accurately identifies a unprecedented violence. Mobs, including supporters
flaw in the argument made in the passage? of the Pakistan Tehreek-e-Insaf party, targeted army
(A) The argument assumes that the shift towards and paramilitary installations, which is a departure
alternative voting methods is solely responsible from the public's usual fear of the army. This has
for the decline in citizens' appreciation of their raised speculation about a potential "Pakistan
role in a democracy. Spring," although its success and impact on the
(B) The argument overlooks the fact that the country's future remain uncertain. The status quoists
momentousness of voting occasions can be may exploit the ongoing economic crisis to suppress
intimidating for some citizens, leading to their the movement. The events unfolding in Pakistan,
disengagement from the democratic process. along with the humanitarian crisis in Afghanistan,
(C) The argument fails to consider that widespread are causes for concern
participation in prominent elections may not While some believe that the Pakistan army will
necessarily translate into a better understanding remain united against the hostility, others argue that
of democratic values by the citizens. Imran Khan, who previously benefited from the
(D) The argument assumes that the confluence of army's support, has tapped into anti-army sentiment
democratic values during election time is the for his own opportunistic agenda. They suggest that
only effective way to shape citizens' attitudes taking firm action against Khan could restore the
toward democracy and their role within it. army's image as a unifying force and bring an end to
the chaos.
5. Which of the following, if true, would most weaken The changing dynamics in Pakistan reflect the
the argument that the experience of voting and the broader geopolitical reality. Since the US troop
optics of elections play a significant role in shaping withdrawal, the army has faced increasing threats.
citizens' attitudes toward democracy and their role Tehreek-e-Taliban Pakistan (TTP) and Baloch
within it, as presented in the passage? groups have launched numerous attacks on the
(A) Recent studies have shown that citizens' armed forces, with the TTP expanding its influence
attitudes toward democracy are primarily in Balochistan and Punjab. Clashes with the Taliban
shaped by their interactions with political along the Afghanistan border have also intensified.
leaders and participation in political campaigns, The army's involvement in the China-Pakistan
rather than the act of voting itself. Economic Corridor (CPEC) has contributed to
(B) Surveys indicate that citizens who vote via public resentment against Chinese investments,
mail-in ballots or early voting methods have particularly in Khyber Pakhtunkhwa and
similar levels of engagement and appreciation Balochistan. However, the army's political weakness
for democracy as those who vote at precinct and security challenges make it unlikely to engage
polling places, suggesting that the experience of in further political misadventures.
voting is not a determining factor in shaping Imran Khan's actions have further undermined the
citizens' attitudes. army's stature. Protesters gained access to military
installations, including General Headquarters

[9]
(GHQ), and targeted army patrols and installations (C) Implement economic reforms and address the
in cities. grievances driving the protests to alleviate the
The outcome of this internal conflict is uncertain, crisis.
(D) Allow for peaceful protests and engage in
but the possibility of international intervention to
dialogue with different factions to find a
prevent instability in Pakistan exists. The instability peaceful resolution.
in Afghanistan already affects Pakistan, and further
destabilization could have a spillover effect. 9. Which of the following best represents the main
Military courts, Khan's potential choices, and the argument presented in the passage?
involvement of the ruling PDM party and higher (A) The protests and violence in Pakistan have
judiciary indicate that the battle has just begun, and raised concerns about a potential "Pakistan
Spring" and uncertainty about the country's
its resolution may depend on the actions of the
future.
people, global powers, and events in Afghanistan. (B) Imran Khan's arrest and the targeting of army
and paramilitary installations indicate a
6. Which of the following is a key factor contributing significant shift in public sentiment towards the
to the uncertainty surrounding the outcome of the military in Pakistan.
internal conflict in Pakistan? (C) The changing dynamics in Pakistan, along with
(A) The potential for international intervention to the humanitarian crisis in Afghanistan, call for
vigilance and potential international
prevent instability in Pakistan.
intervention to prevent further destabilization.
(B) Imran Khan's previous alliance with the (D) The economic crisis and grievances driving the
Pakistan army. protests in Pakistan require the implementation
(C) The public resentment against Chinese of economic reforms to restore stability and
investments in Khyber Pakhtunkhwa and address the underlying issues.
Balochistan.
(D) The ongoing security challenges faced by the 10. Which of the following options presents a paradox
army, including attacks by the Tehreek-e- based on the information provided in the passage?
Taliban Pakistan (TTP) and Baloch groups. (A) The protests in Pakistan reflect a departure
from the public's usual fear of the army, yet the
7. Which of the following, if true, would most weaken arrest of Imran Khan has the potential to restore
the argument presented in the passage that the the army's image as a unifying force.
changing dynamics in Pakistan reflect a broader (B) The changing dynamics in Pakistan and the
geopolitical reality and that the outcome of the humanitarian crisis in Afghanistan call for
internal conflict is uncertain? vigilance, yet international intervention to
(A) The protests in Pakistan were largely peaceful prevent further destabilization is unlikely.
and did not result in significant violence or (C) The violence and targeting of military
disruption. installations in Pakistan indicate a significant
(B) Imran Khan's arrest was supported by a shift in public sentiment towards the military,
majority of the political parties in Pakistan yet the outcome of the internal conflict remains
(C) The neighboring countries have expressed their uncertain.
commitment to non-interference in Pakistan's (D) The economic crisis and grievances driving the
internal affairs. protests in Pakistan require economic reforms
(D) The internal conflict in Pakistan has been to restore stability, yet implementing such
effectively resolved, leading to a period of
reforms could exacerbate the ongoing
stability and unity.
instability.
8. Based on the information provided in the passage,
which of the following courses of action would be PASSAGE III
most appropriate to address the situation in By holding the meeting of the ―G-7‖, or the so-
Pakistan? called group of the world‘s most industrialised
(A) Increase military presence and engage in a nations, in Hiroshima, Japanese Prime Minister
stronger crackdown on protests to restore
Fumio Kishida, who belongs to the city, wanted to
stability.
(B) Seek international intervention to resolve the send out a message of global solidarity for peace.
internal conflicts and prevent further The G-7 members issued a separate statement on
destabilization. Ukraine, hitting Russia with more sanctions, but
[10]
failed to highlight a path towards dialogue and 12. Which of the following, if true, would most weaken
ending the war. Despite the invitation of countries the argument that the G-7's limited representation
such as India, Brazil, Indonesia and Vietnam with a hinders its effectiveness in addressing global
challenges?
less black-and-white view of the world, and Mr.
(A) The G-7 summit in Hiroshima resulted in
Kishida making pertinent references to the views of significant commitments from member
the Global South, the summit‘s statements reflected countries to address global challenges.
a much more polarised view of the world — that of (B) The G-7's exclusion of certain countries is a
G-7 members alone. deliberate strategy to ensure a more focused
If the G-7 grouping wishes to broaden its prism, it and effective decision-making process.
must recognise that it is grossly unrepresentative of (C) The G-20 summit in Delhi is expected to have
the world today. While members together represent limited participation from developing nations,
more than half the world‘s net wealth, the G-7 similar to the G-7 summit.
accounts for less than a third of the global GDP, and (D) The G-7's limited representation does not
just over a tenth of the world‘s population. Apart hinder its ability to collaborate with other
from Japan, the G-7 membership comprises an international organizations to address global
essentially Euro-American worldview, and is not challenges effectively.
discussing expanding that view soon. It has actually
contracted, after it expelled Russia over its 13. Which of the following assumptions is necessary for
annexation of areas of Georgia in 2008 and then the argument that the limited representation of the
Crimea in 2014. It is also hard to justify an G-7 hinders its effectiveness in addressing global
economic grouping that does not include some of challenges?
the world‘s largest economies (China and India) or (A) The perspectives and inputs of countries
the fastest growing GDPs, or biggest global energy outside the G-7 are crucial for understanding
providers. While some efforts were made in and effectively addressing global challenges.
Hiroshima to recognise the G-7‘s role in, for (B) The G-7 countries have not been successful in
example, promoting transparent financing and debt addressing global challenges in the past due to
sustainability for the developing world, or in their limited representation.
compensating for the developed world‘s (C) The exclusion of certain countries from the G-7
contribution to global warming and greenhouse gas significantly impacts the diversity of
emissions, the summit failed to propose concrete perspectives and expertise in addressing global
measures to help defray these responsibilities. As challenges.
the spotlight shifts to the G-20 summit later this year (D) The G-20, with its broader representation, has
in Delhi, it is hoped the grouping will work towards proven to be more effective in addressing
a more inclusive outlook and help in building a global challenges compared to the G-7.
more comprehensive global consensus on some of
the bigger challenges the world faces today. 14. Which of the following courses of action is most
supported by the information presented in the
passage?
11. Based on the passage, which of the following (A) The G-7 should immediately expand its
statements can be inferred? membership to include China and India to
(A) The G-7 summit in Hiroshima aimed to address ensure a more representative global outlook.
the concerns of the Global South countries. (B) The G-7 should focus on promoting transparent
(B) The G-7's exclusion of Russia and limited financing and debt sustainability for the
representation raises questions about its ability developing world to address the imbalances in
to effectively address global challenges. the global economic landscape.
(C) The G-7 countries have made significant (C) The G-7 should work towards reinstating
progress in addressing the economic concerns Russia's membership to enhance its credibility
of developing nations. in addressing global challenges.
(D) The G-20 summit in Delhi is expected to be (D) The G-20 summit should prioritize building a
more inclusive and comprehensive in more comprehensive global consensus on major
addressing global challenges than the G-7 challenges, considering its broader
summit. representation.

[11]
15. Which of the following most accurately captures the department was on, it was abusive toward the other.
main argument presented in the passage? Analysts held that continental philosophy was not
(A) The G-7's limited membership undermines its philosophy at all, but meaningless yet relativistic
babble, something of substantially less than no
ability to address global challenges effectively
value. Continentalists characterised analytic
and hinders the representation of diverse philosophy as useless punctilious logic-chopping
viewpoints. and scientism for its own sake, with no possibility of
(B) The G-7's exclusionary approach in cultural critique or even meaningful connection to
membership selection promotes a Euro- human life as it is actually conducted.
American worldview and perpetuates polarized It is not surprising, however, that the lines of
views of the world. discourse had more in common than the participants
in the ridicule thought they did. Analytic and
(C) The G-7's focus on economic influence and
continental philosophy emerged at the same time in
political stability in membership criteria the Western academy, out of a shared intellectual
neglects the perspectives and interests of history. The rivalry was as professional as it was
countries from the Global South. conceptual, and the contest was always to see which
(D) The G-7's smaller population and GDP side could get rid of the others‘ professors. But in a
representation compared to its wealth thousand ways through the whole century, they were
representation diminishes its influence on embedded in the same zeitgeist. They had a lot of
the same obsessions, as well as a lot of the same
global economic decisions and calls for a more
drawbacks, even if, by 1967, they also had entirely
inclusive outlook. different vocabularies.

PASSAGE IV 16. What is the main idea conveyed in the passage?


‗There is nothing outside the text,‘ wrote Jacques (A) The convergence on questions of language in
Derrida in 1967. Like most everything Derrida said, 20th-century philosophy.
this notorious declaration becomes more difficult to (B) The rivalry and animosity between analytic and
interpret as one examines its context and the context
continental philosophy.
of its context. But it aptly captures the flavour of
academic philosophy at the time it appeared, which (C) The influential role of Jacques Derrida in
was also the year of Richard Rorty‘s anthology The academic philosophy.
Linguistic Turn, which embodied an argument that (D) The shift towards linguistic philosophy in the
the most important philosophy of the 20th century 20th century.
was linguistic philosophy. By then, everyone but a
few reactionaries would have agreed with that 17. Which assumption is implicit in the passage?
assessment. Philosophy had for decades been
(A) Analytic and continental philosophy have no
relentlessly emphasising the nature of language.
There was some dispute about whether there could common ground or shared intellectual history.
be any genuine philosophical questions that were (B) Linguistic philosophy is the most important
not questions about language. branch of philosophy in the 20th century.
Looking back on it from here, the convergence on (C) Both analytic and continental philosophy have
questions of language – indeed, the relentless, limitations and drawbacks.
almost-exclusive focus on it as central to our (D) The participants in the ridicule of the other side
experience, by thinkers otherwise so different that
fully understood the similarities between
they could not or did not care to enter into dialogue
– seems remarkable. It is one of the signal aspects of analytic and continental philosophy.
20th-century intellectual history and a useful lens
through which to view the development of 18. What flaw can be identified in the reasoning
philosophy during that time. presented in the passage?
In the 20th century, Western philosophy split into (A) The passage does not provide enough evidence
two discourses, each with its own canon and jargon,
to support the claim that analytic and
usually referred to as ‗analytic‘ and ‗continental‘.
Mastering them simultaneously (getting a solid continental philosophy emerged at the same
handle on both Martin Heidegger and Bertrand time.
Russell, for example, or both Willard Van Orman (B) The passage assumes that the convergence on
Quine and Michel Foucault), was a very questions of language is the sole defining
intimidating prospect, and few had the motivation. aspect of 20th-century intellectual history.
Almost certainly, if one was housed, one was (C) The passage overgeneralizes the perspectives of
housed in a department that did only one or the
analytic and continental philosophy, failing to
other. And almost certainly, whichever side the
[12]
acknowledge the diversity within each music – any feature, or combination of features, that
discourse. stands out and sticks in a person‘s memory. A hook
(D) The passage fails to address the impact of is supposed to be heard and noticed easily. For
linguistic philosophy on other branches of example, the intended hook in ‗Who Let the Dogs
philosophy, such as the nature of reality, Out‘ is obvious: it‘s the bit where Baha Men chant:
goodness, or beauty. ‗Who let the dogs out? Who, who, who, who?‘
Whether you like the song or not, there is something
19. Which statement would strengthen the argument in this part of it – its rhythm, its simplicity, the
presented in the passage? sound of the voices, or perhaps its repetitiveness –
(A) Linguistic philosophy was widely embraced by that seems to transcend personal taste. It is easily
all philosophers in the 20th century. remembered to the extent that it can enter into your
(B) The rivalry between analytic and continental consciousness and stay there, whether you like it or
philosophy hindered the progress of philosophy not (making it what psychologists call an
as a discipline. ‗earworm‘).
(C) The convergence on questions of language led Much of modern pop could be described as a hook-
to groundbreaking insights and advancements delivery device: ‗Bad Romance‘ by Lady Gaga or
in philosophical thought. ‗Shake It Off‘ by Taylor Swift, for example, are
(D) Jacques Derrida's declaration about the packed full of musical moments that stand out to the
centrality of language in philosophy was widely listener and are easily remembered. Jay Brown, the
contested and disregarded. CEO of the Roc Nation entertainment company,
once suggested that modern pop songs need a hook
20. Which inference can be made based on the in every section to hold a listener‘s attention.
information provided in the passage? Producers certainly seem to know this. From the
(A) Linguistic philosophy completely replaced ‗rah rah‘ and ‗ooh la la‘ vocables in ‗Bad Romance‘
other branches of philosophy in the 20th to the part in ‗Shake It Off‘ when the saxophone line
century. kicks in, the plan appears to be, essentially: if one
(B) The linguistic turn in philosophy sparked a new hook doesn‘t get you, another will.
era of interdisciplinary dialogue. So what makes a part of a song a hook? Well, a
(C) The focus on language in philosophy had little hook is not just a musical moment; it is a musical
impact on cultural critique or practical moment that is interacting with the listener‘s mind.
applications. In particular, it is a product of the interaction
(D) Understanding both analytic and continental between the music and the listener‘s capacities for
philosophy was a rare achievement due to their attention and memory.
divergent vocabularies and perspectives.
21. Which option best summarizes the main idea of the
PASSAGE V passage?
It‘s the ‗yeah, yeah, yeah‘ in the Beatles‘ ‗She Loves (A) Hooks in music are memorable and attention-
You‘, the guitar riff in Blur‘s ‗Song 2‘, or the grabbing moments that stick in a person's
ascending vocal in ‗Kill Bill‘ where SZA sings: ‗I memory.
might kill my ex.‘ For many listeners, each of these (B) Pop music relies heavily on hooks to capture
serves as a hook: a musical or lyrical moment that listeners' attention.
both stands out and is easily remembered. This (C) Hooks can be found in various forms of music,
definition, based on one put forth in the 1980s, is a from pop to classical.
useful way to think about what hooks can be. Pop (D) Hooks are musical elements that interact with the
music is full of hooks, but there are plenty of them listener's mind and memory.
in other kinds of music, too – whether it‘s the motif
at the start of Beethoven‘s 5th Symphony (da-da-da 22. Which option can be inferred as an assumption
DAH), or Bill Evans‘s descending piano chords at underlying the discussion of hooks in music?
the beginning of Miles Davis‘s version of ‗On Green (A) Hooks are intentionally created by musicians to
Dolphin Street‘. A hook can come in many forms, make their songs more popular.
from a particular beat or series of notes to the lyrics (B) All listeners have the same reaction and
or other elements at a certain point in a piece of perception of hooks in music.

[13]
(C) Hooks play a significant role in making a song (D) Hooks in music are designed to cater to
memorable and appealing to listeners. individual preferences and personal taste.
(D) The concept of hooks in music originated in the
1980s. 25. Which option must be true based on the information
provided in the passage?
23. Which option can be inferred from the passage? (A) All listeners remember and appreciate the same
(A) Hooks in music are solely based on the lyrics hooks in music.
and vocal elements. (B) Hooks in music are limited to pop songs and
(B) All pop songs require hooks in every section to cannot be found in other genres.
hold listeners' attention. (C) Hooks are musical moments that interact with
(C) Hooks have the ability to transcend personal the listener's attention and memory.
taste and stick in a person's memory. (D) Hooks in music are solely defined by catchy
(D) Hooks in classical music are primarily based on lyrics and memorable melodies.
complex melodies and harmonies.

24. Which option strengthens the argument presented in


the passage?
(A) Many successful pop songs rely heavily on
hooks to capture and retain listeners' attention.
(B) Hooks in music are solely dependent on the
skill and creativity of the musicians.
(C) All listeners have the same level of attention
and memory capacity when it comes to hooks.

[14]
GK and Current affairs
PASSAGE I States, who voted against this shameful resolution
India abstained in the U.N. General Assembly on a that seeks to isolate and demonise the Jewish state.
resolution that asked the International Court of Lauder further said the ―Measure is a direct
Justice for its opinion on the legal consequences of outgrowth of the biased Commission of Inquiry on
Israel‘s ―prolonged occupation‖ and annexation of Israel, whose commissioners have made anti-
the Palestinian territory. semitic comments and who have been unabashed
The draft resolution ‗Israeli practices affecting the critics of Israel. Referral of this issue to the ICJ is
human rights of the Palestinian people in the yet another barrier to dialogue between Israelis and
Occupied Palestinian Territory, including East Palestinians.‖
Jerusalem‘ was adopted by a recorded vote on World Jewish Congress is an international
Friday, with 87 votes in favour, 26 against and 53 organisation representing Jewish communities in
abstentions, including by India. more than 100 countries to governments,
The resolution decided to request the UN's highest parliaments and international organizations.
judicial body to ―render an advisory opinion‖ on
―what are the legal consequences arising from the 1. When did Israel occupy Gaza Strip and the West
ongoing violation by Israel of the right of the Bank, replaced by (X) in the passage?
Palestinian people to self-determination, from its (A) 1962 (B) 1967
prolonged occupation, settlement and annexation of (C) 1970 (D) None of the Above
the Palestinian territory occupied since (X),
including measures aimed at altering the 2. Which was one of the countries which voted against
demographic composition, character and status of the resolution accompanied by Israel, replaced by
the Holy City of Jerusalem, and from its adoption (Y) in the passage
of related discriminatory legislation and measures.‖ (A) United States of America
It also asked the Assembly ―how do the policies (B) India
and practices of Israel… affect the legal status of (C) Russia
the occupation, and what are the legal consequences (D) Australia
that arise for all States and the United Nations from
this status?‖ 3. When was the International Court of Justice
The (Y) and Israel voted against the resolution Established?
while Brazil, Japan, Myanmar, and France were (A) 1940 (B) 1945
among those that abstained. (C) 1955 (D) None of the Above
Before the vote, Israel's Ambassador to the United
Nations Gilad Erdan said that the ―outrageous 4. Which of the below following facts are true about
resolution‖ calling for the advisory opinion of the the stand about Israel?
International Court of Justice is a ―moral stain on (A) It was stated that holding a vote on Israel on
the U.N. and every country that supports it. No Shabbat is another example of the United
international body can decide that the Jewish Nations' "moral decay," which prevents Israel's
people are ―occupiers‖ in their own homeland. Any position from being heard in a vote whose
decision from a judicial body which receives its outcome is predetermined.
mandate from the morally bankrupt and politicised (B) During the United Nations General Assembly
UN is completely illegitimate.‖ High-Level Week in September 2021,
Mr. Erdan added that the decision to hold a vote Palestinian Authority President Mahmoud
that deals with Israel on Shabbat is another example Abbas stated in his remarks that if Israel did
of the ―moral decay‖ of the U.N., which prevents not withdraw to the 1967 lines within a year,
Israel's position from being heard in a vote whose the Palestinians would turn to The Hague.
results are predetermined. Hence, it was stated that today's vote fulfills
Following the vote, World Jewish Congress Abbas' ultimatum.
President Ronald S. Lauder said in a statement that (C) Both Option A and Option B are true.
the vote at the United Nations exemplifies an (D) None of the above options are true.
ongoing pattern of bias against Israel, and ―we
applaud those 26 countries, including the United
[15]
(C) Scope for Mainstreaming Ayurveda Research
5. Which of the below following people is the in Health Associations
representative of India at United Nations? (D) Scope for Mainstreaming Ayurveda Research
(A) Syed Akbaruddin in Teaching Professionals
(B) Nirupam Sen
(C) Hardeep S Puri 7. Which of the below following components is not a
(D) Ruchira Kamboj part of Ayush?
(A) Ayurveda
(B) Naturopathy
PASSAGE II
(C) Hospitality
National Commission for Indian System of (D) All of the Above components are under
Medicine (NCISM) and Central Council for AYUSH.
Research in Ayurvedic Sciences (CCRAS) under
ministry of Ayush on Monday launched ‗(X), 8. Who is the current minister of AYUSH?
SMART‘- scope for mainstreaming ayurveda (A) Prahaldh Joshi
research in teaching professionals programme. (B) Narendra Modi
(C) Dharmendra Pradhan
―The programme aims to boost scientific research
(D) Sarbananda Sonowal
in healthcare research areas through ayurveda
colleges and hospitals," the Ministry of Ayush said 9. Which of the facts are true about the SMART
in a statement. Scheme?
The programme was launched by NCISM, (A) SMART aims to identify, support and promote
chairman, Vaidya Jayant Deopujari and CCRAS, innovative research ideas in healthcare
director general, Rabinarayan Acharya in the research areas including Osteoarthritis, Iron
presence of NCISM, president, board of ayurveda, Deficiency Anaemia, Chronic Bronchitis,
BS Prasad and other senior officials. Dyslipidemia, Rheumatoid Arthritis, Obesity,
While talking about the initiative, NCISM, Diabetes Mellitus, Psoriasis, Generalised
chairman, Vaidya Jayant Deopujari said, ―I am Anxiety Disorder, Non-alcoholic fatty liver
confident that this program has the potential to disease (NAFLD).
transform clinical research in Ayurveda. It was (B) The program will motivate doctors for taking
observed that the research potential of the large up projects in designated areas of healthcare
community of Ayurveda teachers remains under research and create a large database.
utilised mostly. Therefore, the ‗SMART‘ program (C) Both A and B are true
will have a deep long term rejuvenating impact on (D) Neither A nor B is true
research in the field of Ayurveda and it will be a
great service to the nation, I congratulate CCRAS 10. Which of the following facts are true about
for this initiative and ensure all support on behalf of National Ayush Mission?
NCISM." (A) The scheme involves expansion of the
Speaking about the SMART programme, NCISM, AYUSH sector to promote holistic health of
president, Board of Ayurveda, BS Prasad said, ―The Indians.
large network of Ayurveda colleges and hospitals (B) He was launched in 2014 by the Department of
across the country is an asset for the country in AYUSH under the Ministry of Health and
terms of its healthcare needs. This network has not Family Welfare, during the 12th Plan for
only been offering healthcare services in hardest implementation through States/UTs
times, but it also has contributed significantly in (C) The scheme involves expansion of the
terms of healthcare research in the country. AYUSH sector to promote holistic health of
Indians.
6. Correctly abbreviate the term "SMART" replaced (D) All of the Above
by (X) in the passage
(A) Scope for Maintaining Ayurveda in Research PASSAGE III
Technologies Taliban representatives met with Indian and
(B) Scope for Maintaining Ayurveda Record in Pakistani special envoys and officials amongst a
Teaching Professionals number of international diplomats this week, in an
effort by the Norwegian Government to break the
[16]
impasse in talks on the sidelines of a peace 11. What kind of initiatives can India continue
conference in Oslo. The talks, which came in for supporting in Afghanistan despite the Taliban
criticism from Afghan diaspora groups, including in takeover?
Norway, for ―legitimising‖ the insurgent group that (A) Development Initiatives that benefit
took control of Kabul in August 2021, are the first Afghanistan
time India and Pakistan were invited to a European (B) Military initiatives against the Taliban
country for the talks, although they have been part (C) Diplomatic initiatives to engage with the
of similar efforts at the Moscow format hosted by Taliban
Russia, and in Doha hosted by Qatar. ―Isolating (D) Initiatives that benefit only citizens in
Afghanistan now will be unfortunate, both for the Afghanistan
Afghan people and for us. It could worsen the
situation for the Afghan people and it could lead to 12. What is the Oslo process?
terrorist groups such as IS (Islamic State) building (A) A peace process aimed at resolving the Israeli-
up in the country,‖ Norwegian Foreign Minister Palestinian conflict
Anniken Huitfeldt told local media, confirming that (B) A diplomatic initiative to resolve the Syrian
three Afghanistan civil servants, and not ―top civil war
leadership‖ were participating in the Oslo Forum (C) Series of negotiations between Iran and the
and speaking about the ―major challenges‖ in the P5+1 countries
country. Afghanistan faces a humanitarian crisis (D) An international effort to promote peace
with acute food shortages, and continuing human
rights violations including denial of education and 13. What should India do to handle the situation in
employment to females by the Taliban. Afghanistan?
However, Afghan diplomats opposed to the Taliban (A) Adopt a balanced approach in its dealings with
rule said that Norway was using dialogue and Afghanistan
conflict resolution as an ―excuse‖ to engage with (B) Provide support and aid from Afghanistan
the Taliban, which hasn‘t kept its promises. (C) Align itself completely with Afghanist
―Norway must end these failed efforts and instead (D) Development initiatives for the people of
work multilaterally to help rationalize the Afghanistan
dangerous status quo in Afghanistan. This begins
with steps to resume intra-Afghan talks for a 14. What is the name of the peace conference held in
sustainable political settlement that forms an Oslo to break the impasse in talks between the
inclusive government acceptable to all Afghans, ― Taliban and international diplomats?
said Ashraf Haidari, the Afghan Ambassador to Sri (A) Oslo Accords
Lanka appointed by the previous government. The (B) Oslo Summit
Taliban representatives present declined to (C) Oslo Peace Forum
comment on the meetings. ―There‘s no moral (D) Oslo Process
dilemma with speaking to the Taliban because they
do run the country...If you just boycott or ignore 15. What is the name of India's development initiative
them, nothing is going to be resolved,‖ Michael in Afghanistan?
Vatikiotis, a senior advisor at the Centre for (A) New Silk Road
Humanitarian Dialogue that co-hosted the Oslo (B) SAGAR Mala
Forum told The Hindu, when asked about the (C) Chabahar Port Project
criticism. Among those who spoke at the event held (D) Reconstruction Opportunity Zones
under Chatham House rules, which focussed on a
number of global conflicts, were Norwegian Prime PASSAGE IV
Minister Jonas Gahr Støre, Indonesian Foreign Communication and transactions in the digital
Minister Retno Marsudi, Colombian Foreign ecosystem are fuelled by the collection and
Minister M. Álvaro Leyva Durán; Prosecutor of the processing of enormous amounts of ‗personal data‘
International Criminal Court Karim A.A. Khan KC; of billions of users and even non-users. It is further
and Bangladesh‘s State Minister for Foreign Affairs classified as ‗Personally Identifiable Information‘
Shahriar Alam. (PII) and ‗Personally Sensitive Information‘ (PSI),
even as boundaries blur. Notwithstanding the

[17]
country of incorporation or principal operations, an 17. What is the name of the committee appointed by
entity can be fined under GDPR up to ‗four per cent the Indian government in August 2017 for data
of global revenue‘ for significant harm due to loss, protection?
breach, or misuse of personal data of European (A) Justice K. S. Puttaswamy Committee
residents. Actual fines can be as little as €2,000 or (B) Personal Data Protection Committee
even the record €1.2 billion fine on Meta last week. (C) Digital India Committee
Small hospitals and even football clubs are listed (D) B.N. Srikrishna Committee
alongside marquee names like Google, WhatsApp,
British Airways and Marriott. Most BigTech 18. What is the name of the Chinese law that came into
companies are headquartered in the US and effect in November 2021 to prevent the misuse of
frequently transfer the personal data of users in personal data?
other countries to the US for storage and (A) Personal Information Protection Law (PIPL)
processing. After the 2015 invalidation of the long- (B) Data Security Law (DSL)
standing Safe Harbor agreement between the EU (C) General Data Protection Regulation (GDPR)
and the US, ‗Privacy Shield‘ was formalised in (D) Electronic Communications Privacy Act
2016 but even that was found inadequate in 2020.
Following the 2022 in-principle agreement, the new 19. What is the name of the Indian board that will be
EU-US framework is yet to be concluded established to adjudicate non-compliance with the
Being a major data fiduciary and processor, the provisions of the Personal Data Protection Bill?
government must become a role model in the realm (A) Digital India Board
of data protection. Hence, an independent and (B) Data Protection Board of India
empowered data protection board with (C) Personal Data Protection Authority
parliamentary or judicial oversight is sine qua non, (D) IT Amendment Board
On the other hand, overtly prescriptive and severely
restrictive norms would stifle innovation and 20. What is the name of the Indian act that has privacy
restrict trade by retarding, if not, inhibiting cross- provisions in place under it ?
border data flows. A comprehensive framework for (A) Personal Data Protection Bill
digital governance, however, goes beyond data (B) IT Amendment Act, 2008
protection which is just one piece of the jigsaw (C) Digital India Ac
where it must snugly sit alongside frameworks for (D) Justice K.S. Puttaswamy (Retd) vs Union of
cyber security, competition, artificial intelligence, India
etc. The Indian policies must be cognisant of the
social milieu, political economy, technical prowess, PASSAGE V
and market maturity as well as be compatible with The size of China‘s nuclear arsenal increased from
the business allocation rules, resources and 350 warheads in January 2022 to 410 in January
capabilities. Even the proposed Digital India Act 2023, and it is expected to keep growing, according
may not cover all these aspects. Hence, first, we to estimations by Stockholm International Peace
need a comprehensive national framework for Research Institute (SIPRI). ―Depending on how it
digital governance that in turn should guide and decides to structure its forces, China could
inform all other components — including, but not potentially have at least as many intercontinental
limited to, the data protection law. ballistic missiles (ICBMs) as either the U.S. or
Russia by the turn of the decade, the Stockholm-
16. Which country has a comprehensive data protection based think tank said in its annual assessment
law for the processing of personal data? released on Monday said. The key finding in SIPRI
(A) United States Yearbook 2023 is that the number of operational
(B) China nuclear weapons started to rise as countries‘ long-
(C) India term force modernisation and expansion plans
(D) European Union progressed. ―China has started a significant
expansion of its nuclear arsenal. It is increasingly
difficult to square this trend with China‘s declared
aim of having only the minimum nuclear forces
needed to maintain its national security,‖ Hans M.

[18]
Kristensen, Associate Senior Fellow with SIPRI‘s (D) It indicates that these contries is becoming more
Weapons of Mass Destruction Programme, said in a peaceful
statement issued by SIPRI.
24. What are some potential consequences of India's
India, which has a no-first-use policy for the use of
expansion of its nuclear arsenal?
nuclear weapons and has completed its nuclear triad (A) Increased arms control concerns and regional
with the fielding of two ballistic missile nuclear instability
submarines, is in the process of upgrading its (B) Greater transparency and dialogue between
ballistic missiles. While a submarine-launched nations
intermediate-range ballistic missile is under (C) A decrease in the total global inventory of
development, a new generation ballistic missile, warheads
(D) Straining relations with neighboring countries
‗Agni Prime‘, with a range between 1,000 km-
2,000 km, is close to being inducted, which will 25. What is the total global inventory of warheads
replace older Agni missiles in this range. India has estimated at as of January 2023?
also inducted Agni-5, which has a range of over (A) 12,365 (B) 12,512
5,000 km. The nine nuclear-armed states —the (C) 12,569 (D) 12,986
United States, Russia, the United Kingdom, France,
China, India, Pakistan, the Democratic People‘s PASSAGE VI
Republic of Korea (North Korea) and Israel— In a major milestone, ISRO today successfully hot
continue to modernise their nuclear arsenals and tested 'Cryo CE 20' Engine for a flight duration of
several deployed new nuclear-armed or nuclear- 640 seconds, with the space agency Chairman A S
capable weapon systems in 2022, SIPRI said. Kirankumar describing the feat as a Phenomenal
achievement in the shortest time."
21. What impact could China's potential possession of The test was completed successfully at ISRO
as many ICBMs as the US or Russia have on global Propulsion Complex (IPRC) at Mahendragiri today
nuclear politics ? at 17.15 hrs, an ISRO press release said here. This
(A) It could lead to increased arms control engine is identified for Cryogenic stage hot tests
concerns according to the release. The engine has already
(B) It would lead to greater transparency and undergone two short duration tests and
demonstrated repeatability of engine ignition
dialogue between nations characteristics and steady state performance, it said.
(C) It would reduce the likelihood of nuclear war The test was conducted with Mixture Ratio
Controller (MRC) in closed loop mode for the flight
(D) It would lead to a decrease in the total global duration of 640 seconds, according to ISRO.
inventory of warheads Kirankumar, in his address to the gathering present
at Mahendragiri, described it as "a phenomenal
22. How might India's focus on longer-range nuclear achievement in the shortest time".
weapons impact its relationship with China?
The Fully indigenous cryogenic engine
(A) It could lead to increased tensions between
India and China development is a major step towards the launch of
(B) It would likely improve relations between GSLV MkIII in December, the release said.
India and China Mahendragiri is in Tirunelveli district of Tamil
(C) It would lead to lesser cooperation between Nadu.
India and China
(D) It would have major impact on the relationship 26. Chandrayan 3 plays a vital role in the booming
between India and China
Indian Space sector. Which of the following is not
23. What is the significance of Russia and the United correct in the context of Chandrayan 3?
States possessing almost 90% of all nuclear (A) It would consist of a lander and a rover similar
weapons? to the Chandrayaan-2 but would not have an
(A) It demonstrates that these countries are orbiter.
committed to increasing their nuclear arsenals (B) The Chandrayaan-3 lander will be equipped
(B) It shows that other countries are interested in with a Laser Doppler Velocimeter (LDV).
developing nuclear weapons (C) Chandrayaan 3 spacecraft is the 3rd lunar
(C) It highlights the global imbalance of power in exploration expedition, outlined by the ISRO.
terms of weapons

[19]
(D) Chandrayan 3 explores the southern part of the 29. What is the use of hot test of CE-20 cryogenic
moon contains larger shadowed areas than the engine?
northern pole, which says possibly has a (A) It will power the lander capacity to specific
permanent source of oxygen.
landing.
27. Which of the following are correctly related to the (B) It will help to ensure the satellite subsystem in
Isro missions? the space environment.
I. Mars Orbiter Mission (MOM), also called (C) The CE-20 cryogenic engine, which will
Mangalyaan, is a Spacecraft orbiting Mars since power the Cryogenic Upper Stage of the
2014. It is India's first interplanetary mission. LVM3.
II.ASTROSAT is the first dedicated Indian Earth (D) None of the above
observatory satellite mission launched by ISRO on
30. S. Somanth is the present Secretary of ISRO, Who
28 September 2015.
was secretary during the mission of Chandrayan 2?
III. ESA-ISRO Synthetic Aperture Radar (NISAR)
(A) S kiran Kumar
is a joint project between ESA and ISRO to co-
(B) Shailesh Nayak
develop and launch a dual-frequency synthetic (C) K sivan
aperture radar satellite. (D) S. somanth
(A) I (B) I, II
(C) II, III (D) II

28. Which specific lunar region has been identified as


the intended landing site for the Chandrayaan-3
mission?
(A) Mare Imbrium
(B) Aitken Basin
(C) Sinus Iridum
(D) Copernicus Crater

[20]
Legal Reasoning
PASSAGE I
The Supreme Court observed that 'common 1. Which of the following are correct according to the
intention' for the purpose of Section 34 IPC can be Supreme Court‘s observation in the above case?
formed at the spur of the moment and during the I. Common Intention can be developed even at the
very spur of the moment.
occurrence itself.
II. Common Intention does not make the co-
For Section 34 of the IPC to apply, there should be perpetrators equally liable.
common intention among the co-perpetrators, III. In normal circumstances, direct evidence of
which means that there should be community of common intention will not be found.
purpose and common design, the bench of Justices (A) I and II are correct
Sanjiv Khanna and Sudhanshu Dhulia said. (B) II and III are correct
In this case, the prosecution case against Gurbachan (C) I and III are correct
(D) All are correct
Singh and others was that they came armed with
'lathi', 'toka', axe, and 'gandasi' respectively, and 2. Which of the following statements is correct in
had beaten and inflicted injuries on one Teja Singh, relation to the difference between common
who died on the spot. The Trial Court convicted intention and similar intention?
them for murder under Section 302 IPC. The High (A) The intention of the accused and co-accused
court partly allowed the appeal filed by Gurbachan can be inferred from the facts and
Singh on the ground that common intention could circumstances of each case.
(B) Under common intention, it is considered that
not be inferred from his conduct, as he was only
all the accused have jointly committed the
armed with 'lathi' and had struck only on the feet of offence themselves and are jointly liable.
Teja Singh. (C) Each accused is liable for the offence he has
In appeal, the Apex court disagreed with this actually committed, if the common intention
finding of the High Court and observed that the cannot be proved.
common intention to inflict injuries and cause the (D) All of the above.
death of Teja Singh, can be gathered from the
3. Why did the High Court allow the appeal filed by
conduct and action of Gurbachan Singh. In this Gurbachan Singh against his conviction by the Trail
context, it observed: Court?
―Section 34 of the IPC makes a co-perpetrator, who (A) Because they were armed with 'lathi', 'toka',
had participated in the offence, equally liable on the axe, and 'gandasi' respectively, and had beaten
principle of joint liability. For Section 34 of the IPC and inflicted injuries on one Teja Singh, who
to apply, there should be common intention among died on the spot.
(B) Because the common intention could be
the co-perpetrators, which means that there should
inferred from his conduct.
be community of purpose and common design. (C) Because he was only armed with 'lathi' and had
Common intention can be formed at the spur of the struck only on the feet of Teja Singh and the
moment and during the occurrence itself. Common common intention could not be inferred from
intention is necessarily a psychological fact and as his conduct.
such, direct evidence normally will not be (D) None of the above.
available. Therefore, in most cases, whether or not
4. Why did the Apex Court disagree with the findings
there exists a common intention, has to be
of the High Court?
determined by drawing inference from the facts (A) Because Gurbachan Singh was armed with
proved. Constructive intention, can be arrived at 'lathi', 'toka', axe, and 'gandasi' respectively.
only when the court can hold that the accused must (B) Because the common intention can be gathered
have preconceived the result that ensued in from the conduct and action of Gurbachan
furtherance of the common intention." Singh.
(C) Because the common intention could not be
Reappreciating the evidence on record, the court
inferred from his conduct, as he was only
observed that, all of the accused, including armed with 'lathi' and had struck only on the
Gurbachan Singh, would be responsible for the feet of Teja Singh.
offence under Section 302 of the IPC, irrespective (D) None of the above.
of the part played by them. Therefore, it allowed the
appeal and restored conviction of Gurbachan Singh
under Section 302 IPC.
[21]
5. Which of the following is not correct regarding getting to know about this, Miona‘s mother moved
‗Common Intention‘ according to Section 34 of to the police station contending that the intercourse
IPC? was criminal in nature and that Moina and Kunal
(A) There must be two or more persons doing a must be put in jail. Decide.
criminal act or omission in order to establish (A) Moina and Kunal consented to engage in
common intention. sexual intercourse and made a conscious
(B) In common intention, each person is liable as if choice by exercising their right to freedom and
he has done the act alone. liberty.
(C) Common Intention is not different from same (B) Moina and Kunal were intending to marry on
or similar intention. attaining majority and engaging in consented
(D) None of the above. sexual intercourse prior to that is not an
offence.
PASSAGE II (C) Moina was a minor and her consent to engage
Statutory Rape is defined in Section 375 of Indian in any kind of sexual intercourse is immaterial
Penal Code, (amendment of 2013) which states that and cannot be used as a defence to counter her
―any male, who does an intercourse with any charges of statutory rape
female who is below the age of 18, with or without (D) Kunal was a major and could righteously
her consent will constitute a Statutory Rape‖. This consent to engage in sexual intercourse despite
offence is created to save the exploitation of the Moina‘s minority and the act was not
minor by the adults since they are incompetent to exploitative in nature.
take decisions for themselves. In this kind of rape,
the adult will get punished if he had any kind of 7. Facts in continuation: Kunal was unaware of the
physical interaction with a woman/girl, in the age Moina‘s age and assumed that she was a major.
of 18 years or below, no matter if she has Moina‘s agreement to marriage and further
voluntarily participated in the act or not. Statutory engaging in sexual intercourse was an indicator that
Rape is always in a shadow as the voice of victims Moina was eligible to do so. Kunal did not force
goes unheard or they will not get any support from Moina into it since Moina initiated the intercourse.
the society and our system. The age of consent and Kunal is arguing that the act was not forced and
the legal age of marriage in India is 18 years old. Moina‘s actions indicated that she was a major. Can
The age of consent is the minimum age at which an Kunal argue the same?
individual is considered legally old enough to (A) It was Moina‘s duty to disclose her age to
consent to participate in sexual activity. The Statute Kunal before engaging in sexual intercourse
for this offence is The Protection of Children and the fact that she chose not to do so shreds
against Sexual offences Act, 2012 (POCSO) but it any criminal liability on Kunal.
is regulated by the Section 375 of IPC. The consent (B) Moina actions and intentions of consenting to
of the victim is immaterial in these types of the sexual intercourse and marrying Kunal in
offences. The aim of statutory rape laws is to the future were not an indicator of the law and
protect young minor females from males who might Kunal should have known the consequences of
exploit them and do not take responsibility by his actions.
giving support for the child. This rationale is to (C) Any expectations and indication of marriage
preserve the marriageability of the minor girl and to does not legalise their actions of running away
stop their unwanted teenage pregnancy. for marriage and engaging in sexual
intercourse.
6. Moina, a 16-year-old girl fell in love with an 18- (D) In addition, the consent for marriage, Moina‘s
year-old boy Kunal and intended to marry him as consent to sexual intercourse shifts the
soon as she turns 18. Moina‘s mother was against criminal liability to her shoulders and she must
their marriage since Moina and Kunal practiced prove that Kunal was aware of her age.
different religions. On facing resistance from her
mother, Moina decided to run away with Kunal and 8. Kusha, a girl and Deepak, a boy belonged to a
marry him on attaining majority. Moina and Kunal backward village in Andhra Pradesh where child
started living in an apartment together. They marriages were still prevalent. Both are 16 years
engaged in sexual intercourse before marriage. On old are forcefully married by their families.

[22]
Furthermore, Kusha and Deepak are emotionally 10. Which of the following statements are in
coerced to consummate their marriage and bear a consonance with the law established in the
male child and start a family. Kapil, a women‘s passage?
rights activist, took the case to the police claiming (A) Since statutory rape is illegalised to curtail
that that act was statutory rape and the accused may forcible child marriages, the law should
be charged accordingly. Decide upon the said facts. enforce stronger penalties on child marriages
(A) Kusha and Deepak both have committed and the minimum age of child marriage must
statutory rape since both the parties were be increased.
minors and were not capable of giving consent (B) The age of consent and the age of marriage
for the act. should be the same so that the issue of
(B) Deepak is criminally liable of statutory rape statutory rape and the child marriage is
since both Kusha and Deepak were 16 years old tackled simultaneously.
and so, law was violated by Deepak. (C) The consent of the minor woman is
(C) Kusha being a minor was incapable to considered immaterial and any engagement
consenting to the act and Deepak‘s act fed by in sexual intercourse is illegal so that single
his family‘s coercion violated the law on minor girl does not have to bear the burden
statutory rape. of raising a child.
(D) Deepak may be liable of statutory rape but Kapil (D) The agenda of illegalisation of statutory rape
being unrelated to the minor woman cannot take is to decrease unwilling and involuntary
the case to the police as his word is immaterial. engagement in sexual activities leading to
pregnancies for women who are eligible to
9. The Marriage Age Bill, 2022 redefines marriage marry.
and decreases the age of marriage from 18 to 16 for
women and retains the marriage age for men at 18. PASSAGE III
Madhav is a major of 21 years and Mira is 16 years Joint liability is a term used for people who have
old. Their intended marriage was originally planned committed an act in pursuance of a common
in 2024 when Mira attains marriage age. However, intention, where each of the persons is liable in the
Madhav and Mira have preponed their marriage to same manner, as this act was done by them alone.
next week with their ages remaining 21 and 16 Section 34 IPC, states the joint liability of partners
respectively. They find the marriage to be proper in crime. The section only provides for the
and legal. After consummation of marriage, Mira constitution of joint liability, not the punishment.
gets pregnant at 17 years of age. Reflect on the This section is only a rule of evidence and does not
legality of their situation. constitute a substantive offence. It provides for the
(A) Madhav and Mira were both eligible to get principle of constructive liability.
married and their consummation of marriage is There are a few principles that guide the application
in consonance with the age limit as prescribed of the concept of common intention. It is necessary
by the new law. that there has been a prior conspiracy relating to
(B) Madhav and Mira could not have preponed the that act. When the offence is proved only on the
marriage since Mira was not eligible to basis of circumstantial evidence, the allegations of
consummate the marriage according to the old common intention cannot be established in the
marriage law. absence of meeting of minds. One of the basic most
(C) Madhav and Mira could not have requirements is that there must be some criminal
consummated the marriage that was preponed act. Such a criminal act must be done by ―several
under the new law and thus, they violated the persons.‖ Such a criminal act however would not
law. include acts which are merely invalid in the law. An
(D) Madhav and Mira were not eligible to get example would be a minor man marrying a minor
married and their act of consummation of woman for the marriage is merely void and not a
marriage was illegal. crime. The common intention doctrine states that
there should be an antecedent to the occurrence. A
clear distinction is made between common intention
and common object is that common intention
denotes action in concert and necessarily postulates

[23]
the existence of a pre-arranged plan implying a (C) Along with A, B is also liable for the offence
prior meeting of the minds, while common object as he stood next to A in a jeep waiting to rescue
does not necessarily require proof of prior meeting A from the crowd.
of minds or pre-concert. Common intention has to (D) None of these
be ascertained in the investigations, however, it is
not necessary that it happens before the occurrence 13. Abhishek, Boru and Chetan conspire to rob Peter‘s
only. It might happen during the occurrence of the house for he had an affair with Abhishek‘s sister.
act as well. The intention is curated at any given When they were committing such robbery, they
point in the action. Usually, an overt act would be were caught and thus they had to run. During such
sufficient to show that there is common intention; running Abhishek was caught by Rathore. He
however the absence of the same also does not stabbed Rathore. Boru was waiting right next to the
vitiate the applicability of common intention. The place of murder on a jeep. After Abhishek stabbed
law requires that the accused must be present on the Rathore, he went away along with Boru in the jeep.
spot during the occurrence of the crime and take On their way they found Chetan running and they
part in its commission; it is enough if he is present took Chetan along with them to run away quicker.
somewhere nearby. Decide
(A) All of Abhishek, Boru and Chetan are liable
11. A, B and C decided to commit burglary. They for murder. The very fact that they escaped
broke into a locked house. However, before they together shows that common intention was
had finished their work, a domestic servant present in all the 3.
appeared from the out-house and started shouting.
(B) Abhishek alone is guilty of murder because
A, B and C left the house and started running away.
They were pursued by a small crowd. A, on being though there was common intention to commit
caught by X, one of the persons pursuing them, the offence of burglary, there was no common
stabbed him with a knife kept at road side and ran intention to commit the offence of murder
away. By the time B and C had disappeared. X died (C) The three had only run to save their lives from
on account of the stab wounds. Later, the police the mob. They acted in self-defence and hence
arrested all the three. They were charged for are not liable
attempted burglary and murder of X.
(D) Abhishek and Boru are liable. Chetan joining
(A) Along with A, B and C are also guilty of
murder because A stabbed X in furtherance of them in their jeep after the murder had been
common intention. committed does not show common intention.
(B) Along with A, B and C are also guilty of
murder because A, at the time of stabbing X, 14. A has romantic attachment with B for the past 25
was acting on behalf of B and C and he wanted years. Both of them want to marry each other but
to save not only himself but B and C as well. their parents are not agreeing to the same. A, along
(C) A alone is guilty of murder because though
with his friends C and D decides to run away with
there was common intention to commit the
offence of burglary, there was no common B and marry in the nearby temple. Decide
intention to commit the offence of murder. (A) Elements of Section 34 are satisfied for there is
(D) None of these. presence of common intention
(B) Elements of Section 34 are not satisfied for
12. Please refer to the facts above. While A was there is nothing showing pre-meditation
stabbing X, B stood next to A in a jeep honking for (C) Elements of Section 34 are satisfied for the
A to come in jeep after stabbing, do a clean job and four should have spoken to their parents
run as soon as possible. Decide the liability of A (D) Elements of Section 34 are not satisfied for no
and B. criminal act has been committed
(A) Along with A, B and C are also guilty of
murder because A stabbed X in furtherance of 15. Please refer to the facts above. A is 20 years of
common intention age and B is 17 years of age. Both of them go and
(B) A alone is guilty of murder because though marry in the nearby temple. Decide
there was common intention to commit the (A) Elements of Section 34 are satisfied for there is
offence of burglary, there was no common presence of common intention
intention to commit the offence of murder (B) Elements of Section 34 are not satisfied for
there is nothing showing pre-meditation

[24]
(C) Elements of Section 34 are satisfied for the observe for one's own good; Necessity could also
four should have spoken to their parents amount to defence to a claim for false
(D) Elements of Section 34 are not satisfied for no imprisonment. The test for deciding whether
criminal act has been committed measures falling short of arrest could lawfully be
taken against individuals was whether there was a
PASSAGE IV reasonable suspicion that an individual was
The tort of trespass to a person has developed as it presenting a particular threat. Self-defence: A
is today as a result of many changes and person, to protect himself from an unruly element
modifications. Interference, however slight with a or any other such person or incidents, can trespass
person‘s elementary civil right to security of on the property to preclude the act from
person, and self-determination in relation to his consummation.
own body, constitutes trespass to the person.
Trespass may be done intentionally, deliberately or 16. Ram and Shyam were playing basketball in a park
negligently. The fundamental principle of plain and and during their game, Shyam threw the ball which
incontestable law is that every person‘s body is hit an old lady sitting on a bench in the park,
inviolate. Any unreasonable interference with a causing her injury.
person without any lawful justification amounts to Can Shyam be held liable for the offence of
trespass on a person. Interference, however slight trespass against the lady?
with a person‘s elementary civil right to security of (A) Yes, as Shyam should have been careful while
person, and self-determination in relation to his playing in a public park.
own body, constitutes trespass to the person. (B) No, he did not have the intention to hit the old
Trespass may be done intentionally, deliberately or lady with the ball.
negligently. The fundamental principle of plain and (C) Yes, as the old lady suffered an injury due to
incontestable law is that every person‘s body is the hitting of the ball.
inviolate. Trespass to a person may be categorised (D) No, as Shyam while playing did not notice
as 1. Assault, which is "any act of such a nature as anyone sitting on the bench.
to excite an apprehension of battery"; 2. Battery, "
intentional and unpermitted contact with the 17. Which of the following statements regarding the
plaintiffs‘ person or anything attached to it and defences to trespass can be inferred from the
practically identified with it" and 3. False passage?
imprisonment, is the "unlawful obstruction or (A) Behaving in a manner which contributes to
deprivation of freedom from restraint of movement one's own loss or injury by the plaintiff can be
The basic idea behind trespass to a person is that used as a defence against trespass by the
every person‘s body is inviolate. A person in order accused.
to establish a suit for trespass to a person needs to (B) Trespassing on the property of another person
prove just one thing whether there was an intention while protecting oneself from an unruly
to commit the trespass or not. An act does not element or any other person is a defense
constitute trespass to a person unless it is done with against trespass
intention. Thus intention is the chief criterion for (C) Necessity cannot be taken up as a defence by
trespass to a person. If there is an intention behind the accused.
committing a trespass then it is actionable per se (D) Both A and B
and the plaintiff need not prove any specific or
particular damage. In negligent commission of 18. Ayush and Neha were walking on the road and
trespass to a person, the plaintiff needs to prove that Neha saw a bus approaching their direction at a
the injuries so complained of are reasonably great speed and pushed Ayush, who was not paying
attention in the opposite direction. However, Ayush
foreseeable. In case of direct trespass or intentional
hit the pole and died. Is Neha liable for trespass?
trespass, proof of actual damage is not necessary (A) Yes, as Neha pushed Ayush intentionally.
but in negligent torts, proof of damage becomes (B) No, as Neha did not know that Ayush would
essential. Defences against trespass to body die due to hitting the pole.
Contributory negligence: In law, behaviour that (C) No, as Neha acted out of necessity.
contributes to one's own injury or loss and fails to (D) Yes, as Neha did not act consciously.
meet the standard of prudence that one should

[25]
19. Which of the following statements can be inferred digital video recorders and/or network video
from the passage? recorders. Importantly, the recording system should
STATEMENT 1: No person acting in good faith be such that the data stored therein is preserved for
can be liable for trespass to a person. 18 months, and said: "If the recording equipment,
STATEMENT 2:Trespass includes apprehension of available in the market today, does not have the
unreasonable interference with one‘s person and capacity to keep the recording for 18 months but for
body. a lesser period of time, it shall be mandatory for all
(A) 1 only states, union territories and the Central government
(B) 2 only to purchase one which allows storage for the
(C) Both 1 and 2 maximum period possible". The top court has
(D) Neither 1 nor 2 rightly recognized the role of State Human Rights
Commissions and directed the setting up of District
20. Ishaan and Hardik are neighbours who do not get level Human Rights Courts as prescribed under
along and are always fighting over different issues. Section 30 of the Human Rights Act. The court said
One day, while arguing with each other Ishaan in case of serious injury and/or custodial deaths, the
threw a glass of cold water at Hardik. injured persons shall be free to complain to the
Can he be liable for trespass? State Human Rights Commission as also to Human
(A) Ishaan's act of throwing water at Hardik was Rights Courts. The constitution of Oversight
intentional and thus, he can be held liable for Committees in accordance with the Supreme Court
trespass. Order dated 03.04.2018 should be done at the State
(B) No damage due to the throwing of water can and District levels. The State Level Oversight
be caused to a person, thus Ishaan is not liable. Committee (SLOC) must consist of (i) The
(C) Both of them are fighting with each other, Secretary /Additional Secretary, Home Department;
therefore Ishaan, alone cannot be held liable. (ii) Secretary /Additional Secretary, Finance
(D) It was a petty fight and thus Ishaan cannot be Department; (iii) The Director General /Inspector
held liable. General of Police; and (iv) The
Chairperson/member of the State Women's
PASSAGE V Commission. It shall be the duty of the SLOC to
‗Who will police the police‘ is a big question for see that the directions passed by this Court are
which the world could not give any answer till carried out. Amongst others, the duties shall consist
today. But SC gave the answer - Cameras can of a) Purchase, distribution and installation of
police the police. The possibility of a watch either CCTVs and their equipment; b) Obtaining the
by the citizen through RTI or judiciary through budgetary allocation for the same; c) Continuous
footage of camera recording will prevent the monitoring of maintenance and upkeep of
phenomenon of hurting the person to confess or CCTVs and their equipment; d) Carrying out
admit a crime. Transparency is the system that inspections and addressing the grievances received.
helps police not to resort to third-degree methods to Similarly, the
extract crime-related information. Supreme Court District Level Oversight Committees should also be
asked the government to fix cameras in every constituted with similar obligations.
possible place where the Police interrogate the
persons. To avoid efforts or difficulties of 21. What can be considered as a rationale behind the
investigation, the police mostly use the easy technic rule formulated by the court that CCTVs should be
of torture to extract the information and then collect installed in places of police interrogation?
evidence. The technology available could be (A) The technology available could be effectively
effectively used for the prevention of violation of used for the prevention of violation of Human
Human Rights by police. The Madras High Court Rights by police.
directed the state to make police (B) CCTV recordings will prevent the police from
transparent towards the family of the victim of torturing a person to make him confess the
custodial violence and also gave specific guidelines crime.
to the judicial magistrates and session judges. (C) The rule was formulated so that it could be
Supreme Court went into details and ordered that ascertained whether the police official is
the CCTV camera footage should be stored in present to fulfil his duty or not.

[26]
(D) Both A and B (C) CCTV camera footage should be stored in
digital video recorders and/or network video
22. Anuj is a Secretary of the Finance Department in recorders according to the Supreme Court.
the oversights committee which is formed at the (D) None of the above
District Level in Rajasthan. He refused to purchase
and distribute the CCTVs when he was asked to do 25. Two friends were doing a research project on the
so saying that it was not his duty to do the same. functioning of a police station. While doing field
Is Anuj's reason justified? research, that many people were beaten up and
(A) No, the reason is unjustified as the oversight tortured while in custody by the police. they filed a
committee does not deal with CCTVs at any complaint that the rules of the Supreme Court are
stage. not being followed in those police stations due to
(B) Yes, the reason is justified as the role of the no CCTVs being in place there.
oversight committee is just to purchase and In light of this, which of the following statements is
distribute the CCTVs. correct?
(C) No, the reason is unjustified because the (A) It is mandatory to install CCTVs at all the
oversights committee has to purchase and entry and exit points of a police station.
distribute the CCTVs. (B) CCTVs have to be installed only in prisons and
(D) No, the reason is unjustified as there is no not in police stations.
requirement for CCTVs at the district level. (C) As per the guidelines of the Supreme Court,
CCTVs are to be installed in places where the
23. Many cases of human rights violations by the police police conduct interrogation and the police
were being reported in X district. Due to this, the station is such a place.
state decided to establish a Human Rights Court at (D) Police stations are not compulsorily asked to
the district level which was objected by many install CCTVs it is up to their discretion.
people saying that there is no authority to form such
a court. PASSAGE VI
In light of this, which of the following statement is In the Criminal law, whoever, intending to take
correct? dishonestly any moveable property out of the
(A) No department other than the High Court has possession of any person without that per­son‘s
the authority to establish a district human consent, moves that property in order to such
rights court. taking, is said to commit theft.
(B) The state government under Section 30 of the Theft is defined under section 378 of Indian Penal
Human Rights Act, has the power to establish Code. Section 378 of Indian Penal Code states:
district human rights courts. Explanation 1.—A thing so long as it is attached to
(C) No department other than the Supreme Court the earth, not being movable property, is not the
has the authority to establish a district human subject of theft; but it becomes capable of being the
rights court. subject of theft as soon as it is severed from the
(D) Respective High Courts of the states can earth.
establish Human Rights Commissions when Explanation 2.—A moving effected by the same act
required. which affects the severance may be a theft.
Explanation 3.—A person is said to cause a thing to
24. Which of the following statement/s is incorrect as move by removing an obstacle which prevented it
per the information provided in the passage? from moving or by sepa-rating it from any other
(A) The family of the injured person is free to thing, as well as by actually moving it.
complain to the State Human Rights Explanation 4.—A person, who by any means
Commission and Human Rights Courts in causes an animal to move, is said to move that
cases of custodial deaths. animal, and to move everything which, in
(B) According to the guidelines by the Supreme consequence of the motion so caused, is moved by
Court, CCTV footage can be stored in any that animal.
recording system wherein data is preserved
forever. Explanation 5.—The consent mentioned in the
definition may be express or implied, and may be

[27]
given either by the person in possession, or by any (B) No, because the consent can be implied and
person having for that purpose authority either Mrs. V thought she has the permission to do
express or implied. so.
(C) No, because Mrs. V did not have any dishonest
26. Mr. X found a gold bracelet at a birthday party at intention behind taking the property, hence it‘s
his friend, Mr. Y‘s house. He thought to steal it. not theft.
The moment he was leaving the party, Mr. Y got to (D) Yes, as Mrs. V has taken glasspot without the
know that he has lost his bracelet. And asked consent of Mr. D
everyone to search it. Mr. X got panicked and
stashed the necklace under the cover of a chair and 29. Mrs. Lata pawned her diamond earing in exchange
fancifully started searching it. Has Mr. X for cash to Tanishq jwells. One day she was going
committed theft? by the Tanishq jewells and when nobody was
(A) No, as he left the necklace in Mr. Y‘s own watching, she took her diamond earing and went to
house home. Considering she was title bearer of the
(B) Yes, as he moved the ring with dishonest earing, has she committed theft ?
intention amounting to theft. (A) No, as it was her earing, she cannot be held
(C) No, because he didn‘t left the party with the dishonest in her intention to take it back.
necklace (B) Yes, though she was the owner of earing but
(D) Yes, because he had dishonest intention of the earing was not in her possession.
owning the necklace without the consent of (C) No, as she was owner of earing and she can
Mr. Y. move her earing as per her will.
(D) Yes, because she had not taken consent of the
27. One night a woodcutter went to Mr. Y‘s jewels before taking her earing.
Farmhouse after crossing seven feet boundary wall
and cut down a sandalwood tree to sell it. When the 30. A meets a bullock carrying a box of treasure. He
logs of Sandalwood tree touched the ground and drives the bullock in a certain direction, in order
due the noise of which, the Watchman awakes and that he may dishonestly take the treasure. Has A
lit the light of farmhouse. The woodcutter ran away committed theft?
leaving the logs of Sandalwood tree. Has the (A) No, A has not committed theft just because he
woodcutter committed theft? driven the bullock in a certain direction.
(A) Yes, as trees are nature‘s gift to everybody it (B) No, as A had not taken anything in his
can‘t be stolen. possession from the bullock.
(B) No, because trees are immovable property and (C) Yes, because A had dishonest intention to take
hence cannot be the subject matter of theft the treasure.
(C) Yes, cutting trees on land of another person (D) Yes, as soon as A moved the bullock in a new
with the intention to sell it is theft direction dishonestly, A has committed theft.
(D) None of the above.

28. Mrs. V in good faith, goes to house of Mr. D and


takes a valuable glasspot she honestly believes to be
the property of Mr. K and returns it to Mr. K, he
accepts it. Has Mrs. V committed theft?
(A) Yes, but actual thief is Mr.K who acceoted the
glaspot without Mr.D‘s consent, by using Mrs.
V‘s good faith.

[28]
Quantitative Techniques
PASSAGE I (C) 9: 4
Directions (1 to 5): Study the following data (D) 4: 9
carefully and answer the questions given below. 5. What is the average marks obtained by Aman,
Three friends Aman, Baman and Chaman are Baman and Chaman in Science?
studying in the same class. Each of them takes five (A) 113.33
subjects English, Hindi, Science, History and Art. (B) 105.5
The college conducted half yearly examination and (C) 107.5
college decided the maximum marks for each (D) 101.33
subject is 150. The average marks obtained by
Aman in all subjects is 100. The percentage of PASSAGE II
marks obtained by Aman in English, Hindi, Science Directions (6 to 10): The following line graph
and History are 60%, 80%, 90% and 50% represent the ratio of selling price and cost price
respectively. Baman scored average marks in all of five different article and table chart represent
the marked price of the article.
subjects 95. The marks obtained by Baman in
Hindi, Science, History and Art are 115, 125, 80
and 70 respectively. The average marks of Chaman
in all five subjects is 80. The average marks
obtained by Chaman in first three subject English,
Hindi and Sciences is 60 and last three subject
Science, history and Art is 110.

1. What is the ratio of marks obtained by Aman in Art


to the marks obtained by Baman in English?
(A) 8: 7
(B) 16 : 17
(C) 6: 7
(D) 7: 9 Article Marked price
A 1250
2. If Chaman scored in history 45 marks more than the B 1580
marks scored by Baman in History, then find the C 1640
percentage of marks obtained by Chaman in D 2450
History. E 2750
(A) 60%
(B) 75% 6. If article C and D sold at the marked price, then the
(C) 80% approximate difference between profit earned on C
(D) 90% and profit earned on D was:
(A) Rs 552
3. If Chaman scored in history 45 marks more than (B) Rs 542
the marks scored by Baman in History then what is (C) Rs 612
the ratio of marks obtained by Baman in Hindi to (D) Rs 632
the marks obtained by Chaman in Art?
(A) 2: 3 7. If the ratio between cost price of article A and E
(B) 11: 15 was 3: 5 and the marked price of article E was Rs
(C) 23: 25 250 more than selling price of article E, then the
(D) 23: 26 selling price of article A was:
(A) Rs 750
4. If Marks obtained by Chaman in Hindi is 60, then (B) Rs 650
what is the ratio of marks obtained by Chaman and (C) Rs 850
Aman in English? (D) Rs 770
(A) 1: 2
(B) 9: 5
[29]
8. If the ratio between selling price of article B and D 10. If the discount percentage on article D and E are
was 3:2 and the discount given on article D was 20% and 10% respectively, then the ratio between
20%, then the cost price of article B was: cost price of article D and E was:
(A) Rs 2460 (A) 2336: 3455
(B) Rs 2140 (B) 3226: 3355
(C) Rs 2260 (C) 3136: 4455
(D) Rs 2450 (D) 3126: 4425

9. If the cost price of article C and E was 50% and


80% less than their marked price. Then the
approximate discount percent given on article C
and D together was:
(A) 67.66%
(B) 65.66%
(C) 72.8 %
(D) 74.42%

[30]
Answer Key
Section I 6. (A) 12. (A) 13. (D)
1. (C) 7. (A) 13. (A) 14. (D)
2. (A) 8. (C) 14. (B) 15. (D)
3. (D) 9. (A) 15. (D) 16. (B)
4. (B) 10. (A) 16. (D) 17. (D)
5. (B) 11. (B) 17. (D) 18. (C)
6. (A) 12. (D) 18. (A) 19. (C)
7. (A) 13. (A) 19. (B) 20. (A)
8. (A) 14. (D) 20. (B) 21. (D)
9. (B) 15. (A) 21. (A) 22. (C)
10. (A) 16. (A) 22. (A) 23. (B)
11. (B) 17. (C) 23. (C) 24. (B)
12. (B) 18. (C) 24. (A) 25. (C)
13. (A) 19. (C) 25. (B) 26. (B)
14. (D) 20. (D) 26. (D) 27. (C)
15. (B) 21. (A) 27. (A) 28. (C)
16. (C) 22. (C) 28. (B) 29. (A)
17. (D) 23. (C) 29. (C) 30. (D)
18. (C) 24. (A) 30. (C)
19. (B) 25. (C) Section V
20. (B) Section IV 1. (B)
21. (A) Section III 1. (C) 2. (C)
22. (C) 1. (B) 2. (D) 3. (D)
23. (B) 2. (A) 3. (C) 4. (D)
24. (B) 3. (B) 4. (B) 5. (A)
25. (A) 4. (C) 5. (C) 6. (B)
5. (D) 6. (C) 7. (A)
Section II 6. (D) 7. (B) 8. (D)
1. (B) 7. (C) 8. (C) 9. (C)
2. (C) 8. (D) 9. (C) 10. (C)
3. (C) 9. (B) 10. (C)
4. (B) 10. (D) 11. (C)
5. (B) 11. (A) 12. (C)

[31]
Hints & Solutions
Section I communication and documentation. The other
PASSAGE I options are mentioned in the passage but are not the
central idea of the passage.
1. Correct Option C
(H & S) PASSAGE II
The passage states that the use of legalese is crucial 6. Correct Option A
to ensure the precision and clarity of legal (H & S)
The author argues that loneliness is a significant
communication and documentation.
health risk that can be as dangerous as smoking. The
passage suggests that governments may not take
2. Correct Option A adequate measures to address this issue, even though
it affects people in both rich and poor countries. The
(H & S)
author also argues that measuring loneliness is
Legalese can minimize confusion and imprecise, and that solitude as a choice should not be
misunderstandings among non-lawyers regarding extinguished. However, the author's main argument
legal concepts. is that loneliness is a significant health risk that may
not receive the attention it deserves from
policymakers.
3. Correct Option D
(H & S) 7. Correct Option A
(H & S)
The passage argues that while the use of legalese is
The passage suggests that measuring loneliness is
necessary for precision and clarity in legal imprecise because variations such as the interview's
communication and documentation, it can also create location and the interviewer's characteristics can
majorly swing responses. In addition, responses may
challenges for non-lawyers. Therefore, statement D,
reveal more about normative desires than what the
which asserts that legalese is necessary to make legal respondent actually feels. The passage does not
communication and documentation accessible to suggest that poverty and geography provide
immunity to loneliness, and it is not stated that
non-lawyers, is not supported by the passage.
loneliness is a subjective experience that cannot be
quantified.
4. Correct Option B
(H & S) 8. Correct Option A
(H & S)
The passage describes legal language as
The passage suggests that even if monies were
"convoluted" which means complicated, intricate, or allocated for schemes that Vivek Murthy
difficult to understand. Therefore, the closest word in recommends, it is not clear how these would sidestep
the "polarisation" that has unhealthily shrunk
meaning to "convoluted" is Option B, Complicated.
discussion networks in the first place. The passage
does not suggest that US healthcare policies do not
allocate sufficient funding for social connection
schemes, that solitude as a choice is not desirable in
5. Correct Option B
the US, or that the US lacks sufficient data on
(H & S) loneliness.
The passage discusses the use of legal language and
its importance in communicating legal concepts and 9. Correct Option B
documents with precision and clarity. The central (H & S)
idea of the passage is that the use of precise and Throughout the passage, the author presents a
unambiguous legal language is crucial to avoid nuanced perspective on loneliness. While
confusion and misunderstandings in legal acknowledging that it is a serious problem with

[32]
significant health consequences, the author also 14. Correct Option D
highlights the challenges of measuring and (H & S)
addressing loneliness. Additionally, the author
Answer: D) He has played with Aaron Rodgers
argues that personal autonomy and choice should be
respected in efforts to combat loneliness. Option b) before. This is implied in the statement "Rodgers
best captures this complexity and ambiguity of the will reunite with his top target in Green Bay from a
author's opinion on loneliness.
year ago, Allen Lazard," indicating that Lazard and
10. Correct Option A Rodgers have played together on the Green Bay
(H & S) Packers team in the past.
The passage takes a critical stance on the issue of
loneliness, noting that while the surgeon general's
advisory may have important implications, it is 15. Correct Option B
unlikely to be acted upon by many countries. The (H & S)
author also points out flaws in measuring loneliness Explanation: In this sentence, the subject (The Jets,
and raises concerns about potential unintended
consequences of addressing the issue. Overall, the along with their draft picks) is singular, and the verb
tone of the passage is somewhat skeptical and (was exchanged) agrees with it in number. This
critical, which aligns with the definition of "cynical." demonstrates proper subject-verb agreement. The
other options have errors in subject-verb agreement:
PASSAGE III
A) should be "Aaron Rodgers and the Jets were
11. Correct Option B
(H & S) making headlines," C) should be "Rodgers, as well
Answer: B) He has had a conflict with his coach in as his teammates, were excited about the trade," and
Green Bay. This is implied in the statement "feud D) should be "Green Bay and the Jets have agreed to
with his coach in Green Bay," indicating that there a deal that sends Rodgers to New York."
has been tension or disagreement between Rodgers
and his coach. PASSAGE IV
16. Correct Option C
12. Correct Option B (H & S)
(H & S) Explanation: The passage states that the deletion of
Answer: B) Because he will bring a lot of attention such knowledge from the syllabi has serious
to the team. This is implied in the statement consequences for the project of shaping the
"Rodgers arrives in the Big Apple just in time to normative character of our society, leading to the
steal New Yorkers‘ attention away," indicating that elimination of discussion and/or persuasion through
Rodgers will bring a lot of media coverage and deliberation of normative concepts such as justice,
public interest to the Jets. equality, and secularism.

13. Correct Option A 17. Correct Option D


(H & S) (H & S)
Answer: A) They had a losing record. This is Explanation: The passage discusses the serious
implied in the statement "The Jets finished 7-10 in consequences of the deletion of normative concepts
2022," indicating that they did not have a winning such as justice, equality, and secularism from
record in the previous season. classroom discussions, which are core constitutional
values.

[33]
18. Correct Option C 23. Correct Option B
(H & S) (H & S)
Explanation: The passage states that the collective The passage clearly mentions that the UGC's draft
consequence of conceptual incomprehension and the guidelines aim to promote research and instruction in
decision to remove certain concepts and chapters Indian Knowledge Systems (IKS), at all levels of
from the classroom altogether tends to produce the education, by preparing and teaching new
same result in the name of rationalization. courses/programmes at the undergraduate and
postgraduate levels. The guidelines picture an
19. Correct Option B amalgamation of fundamental IKS concepts into
(H & S) modern textbooks as well as classroom-based
It argues that the revisions would intensify a teaching–learning activities. Therefore, option B is
different kind of burden on students, create the correct answer. Option A is incorrect as the
conceptual incomprehension and moral-social guidelines do not aim to replace existing modern
burden, and affect the normative character of society. curricula with traditional Indian knowledge systems.
Option C is only a part of the guidelines, and the
20. Correct Option B guidelines are not limited to promoting Indian
(H & S) systems of medicine. Option D is clearly mentioned
"Hamstrung" means impeded or hindered in the to be a rupture in Indian knowledge systems sourced
passage.. from the introduction of Western education, and
therefore, cannot be the reason behind the draft
PASSAGE V guidelines.
21. Correct Option A
(H & S) 24. Correct Option B
Solutions (H & S)
The NEP 2020 mandate is mentioned in the passage, The passage celebrates the National Education
and it states that it is aimed at promoting research Policy (NEP) 2020 mandate to promote research and
and instruction in Indian Knowledge Systems (IKS) instruction in Indian Knowledge Systems (IKS) at all
at all levels of education. levels of education. It highlights the efforts made by
the University Grants Commission to prepare and
22. Correct Option C teach new IKS courses/programs at the
(H & S) undergraduate and postgraduate levels. The passage
Solutions also commends the spread of Indian science,
The guidelines mention the emphasis on highlighting technology, medical pharmacopoeia, and literature to
the continuity of Indian knowledge systems right Asia and Europe from ancient to modern times.
into the 19th century. This can be interpreted as an Overall, the tone of the passage is celebratory of
effort to showcase the long-standing history and Indian knowledge systems and the efforts made to
contributions of Indian knowledge systems. integrate them into modern education.

[34]
25. Correct Option A their role in a democracy. Option B captures the
(H & S) essence of the argument by stating that the practice
The passage discusses the draft guidelines published of popular voting effectively highlights and
by the University Grants Commission to promote the promotes democratic values.
integration of Indian knowledge systems (IKS) into Option A is incorrect because the passage does not
higher education curricula. The guidelines aim to argue that the shift towards alternative voting
incorporate IKS courses/programs at the methods has undermined democratic values. It rather
undergraduate and postgraduate levels and require acknowledges the shift but focuses on the
students to earn at least 5% of their mandated credits importance of the three defining features of popular
from IKS courses. The guidelines highlight the voting.
contributions of early Indians in various fields and Option C is incorrect because the passage does not
emphasize the continuity of the Indian knowledge explicitly mention a decline in citizens' appreciation
tradition. The passage does not suggest that Indian of their role in a democracy due to the increasing use
knowledge systems are superior to Western of postal voting. It emphasizes the significance of
knowledge systems or that the NEP 2020 aims to the three key features of popular voting in shaping
replace Western knowledge systems with Indian citizens' attitudes.
knowledge systems. While the passage does mention Option D is incorrect because the passage
the rupture caused by the introduction of Western emphasizes the importance of rituals of voting and
education, this is not the central idea of the passage. their role in reinforcing the momentousness of the
occasion for participants and witnesses. It does not
Section II argue that such rituals are no longer necessary in
PASSAGE I modern democracies.
1. Correct Option B Hence Option B is the right answer.
(H & S)
Solutions 2. Correct Option C
Answer: B) The practice of popular voting, (H & S)
characterized by formal equality, widespread Answer: C) The confluence of democratic values
participation, and momentous occasions, effectively during election time increases citizens' trust in the
highlights and promotes democratic values. democratic process.
Explanation: Option B accurately summarizes the Explanation: The passage does not provide direct
argument made in the passage. The passage evidence for any of the assumptions mentioned,
highlights three key features that define the making this a challenging logical reasoning question
democratic practice of popular voting: formal that requires careful analysis of the passage.
equality in vote-counting procedures, widespread Option A is incorrect because the passage does not
participation in prominent elections, and the state that the shift towards alternative voting
momentousness of voting occasions. These features methods has resulted in a decline in the number of
are seen as essential in promoting and emphasizing citizens participating in elections. While it
democratic values such as individual dignity, acknowledges the shift, it primarily focuses on the
political equality, and large-scale collective action. features that define the democratic practice of
The passage argues that the confluence of these popular voting.
values during election time helps citizens appreciate

[35]
Option B is incorrect because the passage Option A strengthens the argument by providing
acknowledges that the United States is an outlier in research evidence that citizens who personally
terms of the shift towards alternative voting participate in casting their votes develop a stronger
methods. It does not make a claim about the sense of civic duty and engagement with democratic
universal embrace of the practice of popular voting processes, which supports the confluence of
by all democracies worldwide. democratic values during election time. However,
Option C is the correct assumption based on the option C provides more direct support for the
passage. The passage argues that the confluence of specific aspects of popular voting mentioned in the
democratic values during election time, which argument.
includes formal equality, widespread participation, Option B introduces the concept of strict voter
and momentous occasions, helps citizens appreciate identification requirements, which is not directly
their role in a democracy. It suggests that the addressed in the passage. While it claims an increase
experience of voting and the optics of elections play in voter turnout and public trust, it does not directly
a significant role in shaping citizens' attitudes toward strengthen the argument regarding the defining
democracy and their trust in the democratic process. features of popular voting as described in the
Option D is incorrect because the passage does not passage.
make any explicit claims about the security and Option C is the correct answer as it directly supports
accessibility of postal voting in comparison to other the argument made in the passage. The passage
voting methods. While the passage mentions emphasizes the momentousness and symbolic value
increased interest in postal voting in the US and its of election day in reinforcing citizens' belief in the
long-standing use in Switzerland, it does not provide democratic system. If public opinion surveys
a basis for determining whether postal voting is the consistently indicate that citizens perceive election
most secure and accessible method in modern day as a significant event that holds symbolic value
democracies. and reinforces their belief in the democratic system,
Hence, Option C is the right answer. it strengthens the argument that the practice of
popular voting effectively highlights and promotes
3. Correct Option C democratic values.
(H & S) Option D introduces the concept of convenience
Answer: C) Public opinion surveys consistently voting methods and claims a decline in political
indicate that citizens perceive election day as a apathy. While this may be a positive outcome, it
significant event that holds symbolic value and does not directly strengthen the argument regarding
reinforces their belief in the democratic system, the defining features of popular voting as described
reinforcing the momentousness and large-scale in the passage. Option C provides more direct
collective action aspects of popular voting. support for the argument.
Explanation: In this question, we are looking for the Hence, option C is the right answer
statement that, if true, would provide the strongest
support for the argument that the practice of popular 4. Correct Option B
voting effectively highlights and promotes (H & S)
democratic values, as described in the passage. Solutions
Answer: B) The argument overlooks the fact that the
momentousness of voting occasions can be

[36]
intimidating for some citizens, leading to their similar levels of engagement and appreciation for
disengagement from the democratic process. democracy as those who vote at precinct polling
Explanation: Option B correctly identifies a flaw in places, suggesting that the experience of voting is
the argument made in the passage. The argument not a determining factor in shaping citizens'
emphasizes the momentousness of voting occasions attitudes.
and their role in shaping citizens' attitudes toward Explanation: Option B is the correct answer because
democracy. However, it fails to consider that the it weakens the argument by providing evidence that
same momentousness can be intimidating for some challenges the role of the voting experience in
citizens, leading to their disengagement from the shaping citizens' attitudes toward democracy. If
democratic process. By overlooking this possibility, surveys indicate that citizens who vote via mail-in
the argument presents a one-sided view of the impact ballots or early voting methods have similar levels of
of momentous voting occasions and does not engagement and appreciation for democracy as those
acknowledge the potential negative consequences it who vote at precinct polling places, it suggests that
may have for certain individuals. the experience of voting itself may not be the
Option A is incorrect because the argument does not determining factor in shaping citizens' attitudes as
solely attribute the decline in citizens' appreciation argued in the passage.
of their role in a democracy to the shift towards Option A is not the correct answer because it
alternative voting methods. While it acknowledges introduces a different factor, the interactions with
the shift, it focuses on the defining features of political leaders and participation in political
popular voting rather than assigning sole campaigns, as the primary driver of citizens' attitudes
responsibility for the decline. toward democracy. It does not directly address the
Option C is incorrect because the argument does not impact of the voting experience on citizens' attitudes.
directly address the understanding of democratic Option C is not the correct answer because it
values by citizens. It focuses on the importance of introduces mandatory voting as a factor influencing
the defining features of popular voting and their citizens' attitudes toward democracy. While it
impact on citizens' attitudes and appreciation of their provides an alternative explanation, it does not
role in a democracy. directly challenge the role of the voting experience
Option D is incorrect because the argument does not or the optics of elections.
assume that the confluence of democratic values Option D is not the correct answer because it
during election time is the only effective way to suggests that citizens' attitudes toward democracy
shape citizens' attitudes toward democracy and their are primarily shaped by socio-economic status and
role within it. It emphasizes the significance of education level, minimizing the impact of the voting
popular voting and its defining features in achieving experience. However, it does not directly challenge
this, but it does not exclude other potential factors or the role of the voting experience or the optics of
methods. elections.
Hence, option B is the right answer. In summary, option B weakens the argument by
presenting evidence that citizens who vote via
5. Correct Option B alternative methods have similar levels of
(H & S) engagement and appreciation for democracy as those
Answer: B) Surveys indicate that citizens who vote who vote at precinct polling places, suggesting that
via mail-in ballots or early voting methods have

[37]
the voting experience may not be the determining external influences can significantly impact the
factor in shaping citizens' attitudes. resolution of the conflict, adding to the uncertainty
Hence, Option B is the right answer. surrounding its outcome.

PASSAGE II 7. Correct Option A


6. Correct Option A (H & S)
(H & S) Answer: A) The protests in Pakistan were largely
Answer: A) The potential for international peaceful and did not result in significant violence or
intervention to prevent instability in Pakistan. disruption.
Explanation: Option A is the correct answer as it Explanation: Option A is the correct answer as it
identifies a key factor contributing to the uncertainty weakens the argument presented in the passage. The
surrounding the outcome of the internal conflict in argument emphasizes the significance of the protests
Pakistan. The passage mentions that the possibility and the public's actions against army and
of international intervention to prevent instability in paramilitary installations in reflecting the changing
Pakistan exists. This indicates that external factors dynamics in Pakistan. However, if it is true that the
and the involvement of global powers could play a protests were largely peaceful and did not result in
significant role in shaping the outcome of the significant violence or disruption, it undermines the
conflict, adding to the uncertainty. argument's claim about the extent and impact of the
Option B is incorrect because while the passage internal conflict.
mentions Imran Khan's previous alliance with the Option B is incorrect because the argument does not
Pakistan army and how his actions have undermined rely on the support of political parties for Imran
the army's stature, it does not explicitly state that this Khan's arrest. The focus of the argument is on the
is a key factor contributing to the uncertainty broader implications of the events and the
surrounding the outcome of the conflict. uncertainty surrounding the outcome of the internal
Option C is incorrect because although the passage conflict.
mentions public resentment against Chinese Option C is incorrect because the argument does not
investments in Khyber Pakhtunkhwa and mention the neighboring countries' commitment to
Balochistan, it does not suggest that this factor is non-interference in Pakistan's internal affairs.
directly linked to the uncertainty surrounding the Therefore, this option does not directly weaken the
outcome of the internal conflict. argument presented in the passage.
Option D is incorrect because while the passage Option D is incorrect because the argument asserts
mentions the ongoing security challenges faced by that the outcome of the internal conflict in Pakistan
the army, including attacks by the Tehreek-e-Taliban is uncertain, implying that it has not been effectively
Pakistan (TTP) and Baloch groups, it does not resolved. This option contradicts the argument and
specifically state that this factor contributes to the does not weaken it.
uncertainty surrounding the outcome of the conflict. Hence, option A weakens the argument by
Hence, option A correctly identifies the potential for suggesting that the protests were largely peaceful
international intervention to prevent instability in and did not result in significant violence or
Pakistan as a key factor contributing to the disruption. This challenges the argument's claim
uncertainty surrounding the outcome of the internal about the magnitude and consequences of the events,
conflict. The involvement of global powers and casting doubt on the broader geopolitical

[38]
implications and the uncertainty of the internal suggest that peaceful protests and dialogue are the
conflict in Pakistan. appropriate means to address the situation.
Hence, the correct option is C as the most
8. Correct Option C appropriate course of action would be to implement
(H & S) economic reforms and address the grievances driving
Answer: C) Implement economic reforms and the protests to alleviate the crisis in Pakistan.
address the grievances driving the protests to
alleviate the crisis. 9. Correct Option A
Explanation: Option C is the most appropriate course (H & S)
of action based on the information provided in the Solutions
passage. The passage highlights the protests Answer: A) The protests and violence in Pakistan
triggered by the arrest of former Prime Minister have raised concerns about a potential "Pakistan
Imran Khan and the resulting violence and instability Spring" and uncertainty about the country's future.
in Pakistan. It also mentions the ongoing economic Explanation: Option A best represents the main
crisis and the fact that the protests may be driven by argument presented in the passage. The passage
economic grievances. Therefore, implementing discusses the protests triggered by the arrest of
economic reforms and addressing the grievances that former Prime Minister Imran Khan and the
are fueling the protests would be a logical step to subsequent violence and targeting of military
alleviate the crisis and restore stability in the installations. It mentions the speculation about a
country. potential "Pakistan Spring" and the uncertainty
Option A, increasing military presence and engaging surrounding the country's future as a result of these
in a stronger crackdown on protests, may lead to events. This argument focuses on the concerns and
further violence and escalate the situation rather than uncertainties arising from the protests and their
resolving it. The passage does not suggest that a implications for Pakistan.
stronger military response is necessary or Option B is not the main argument of the passage,
appropriate. although it is mentioned as part of the discussion.
Option B, seeking international intervention, is not The targeting of army and paramilitary installations
supported by the information provided in the is presented as a departure from the public's usual
passage. While it mentions the possibility of fear of the military, but it is not the central point of
international intervention to prevent instability, it the passage.
does not indicate any specific actions or intentions of Option C is also mentioned in the passage, but it is
the international community. Therefore, it is not a not the main argument. While the changing
course of action that can be reliably inferred from the dynamics in Pakistan and the humanitarian crisis in
passage. Afghanistan are discussed, the argument does not
Option D, allowing for peaceful protests and revolve around the need for international
engaging in dialogue with different factions, may be intervention or vigilance. These factors are
a desirable approach in general, but the passage does mentioned as additional causes for concern but are
not provide information to support this specific not the primary focus of the passage.
course of action. It mentions the violence and Option D addresses the economic crisis and
disruption caused by the protests but does not grievances driving the protests, which are discussed
in the passage. However, it does not capture the main

[39]
argument of the passage, which is centered on the Option D does not present a paradox. The passage
protests, violence, and uncertainty about Pakistan's acknowledges the economic crisis and grievances
future. driving the protests and suggests that economic
Hence, option A best represents the main argument reforms are necessary to restore stability. It does not
of the passage, emphasizing the concerns raised by suggest that implementing such reforms would
the protests and violence in Pakistan and the exacerbate the ongoing instability.
resulting uncertainty about the country's future. Hence, option A presents a paradox by highlighting
the contradiction between the protests reflecting a
departure from the public's usual fear of the army
10. Correct Option A and the potential for Imran Khan's arrest to restore
(H & S) the army's image as a unifying force.
Answer: A) The protests in Pakistan reflect a
departure from the public's usual fear of the army, PASSAGE III
yet the arrest of Imran Khan has the potential to 11. Correct Option B
restore the army's image as a unifying force. (H & S)
Explanation: Option A presents a paradox based on Answer: B) The G-7's exclusion of Russia and
the information provided in the passage. The passage limited representation raises questions about its
states that the protests in Pakistan, including the ability to effectively address global challenges.
targeting of military installations, represent a Explanation: The passage suggests that the G-7
departure from the public's usual fear of the army. summit failed to highlight a path towards dialogue
However, it also suggests that the arrest of Imran and ending the war in Ukraine, despite issuing
Khan, which is seen as a potential opportunity to sanctions against Russia. It also mentions that the G-
restore the army's image as a unifying force, could 7 is unrepresentative of the world today and does not
contradict this departure. The paradox lies in the include some of the largest economies, fastest-
contradiction between the protests reflecting a shift growing GDPs, or biggest global energy providers. It
in public sentiment towards the military and the further states that the G-7's contraction by expelling
possibility of the army being seen as a unifying force Russia raises questions about its ability to effectively
through Khan's arrest. address global challenges. Therefore, it can be
Option B does not present a paradox. While the inferred that the G-7's exclusion of Russia and
passage mentions the need for vigilance and the limited representation raises questions about its
potential for international intervention to prevent ability to effectively address global challenges.
further destabilization in light of the changing Option A is incorrect because the passage does not
dynamics in Pakistan and the humanitarian crisis in explicitly state that the G-7 summit aimed to address
Afghanistan, it does not suggest that such the concerns of the Global South countries. While
intervention is unlikely. the passage mentions Prime Minister Kishida
Option C is not a paradox but rather a statement of making references to the views of the Global South,
fact based on the information provided in the it does not indicate that the summit specifically
passage. The violence and targeting of military addressed those concerns.
installations indicate a significant shift in public Option C is incorrect because the passage mentions
sentiment towards the military, while the outcome of that while some efforts were made to recognize the
the internal conflict remains uncertain. G-7's role in promoting transparent financing and

[40]
debt sustainability for the developing world, the Option B suggests that the G-7's exclusion of certain
summit failed to propose concrete measures to help countries is a deliberate strategy to ensure a more
defray these responsibilities. Therefore, it does not focused and effective decision-making process.
suggest significant progress in addressing the Although this information provides a potential
economic concerns of developing nations. rationale for the limited representation, it does not
Option D is incorrect because the passage does not directly weaken the argument's claim that the limited
provide information about the expected inclusivity representation hinders effectiveness.
and comprehensiveness of the G-20 summit in Delhi. Option C mentions that the G-20 summit in Delhi is
While it mentions the hope for a more inclusive expected to have limited participation from
outlook and comprehensive global consensus, it does developing nations, similar to the G-7 summit.
not explicitly state that the G-20 summit will be While this may provide a comparison between the
more inclusive and comprehensive than the G-7 two summits, it does not specifically address the
summit. argument's claim about the G-7's limited
Hence option B is the correct option. representation and its impact on effectiveness.
Hence, option D weakens the argument by
12. Correct Option D suggesting that the G-7's limited representation does
(H & S) not hinder its ability to collaborate effectively with
Answer: D) The G-7's limited representation does other international organizations. This challenges the
not hinder its ability to collaborate with other argument's assertion that the G-7's limited
international organizations to address global representation hinders its effectiveness in addressing
challenges effectively. global challenges.
Explanation:
Option D weakens the argument by stating that the 13. Correct Option A
G-7's limited representation does not hinder its (H & S)
ability to collaborate with other international Answer: A) The perspectives and inputs of countries
organizations effectively. This implies that despite outside the G-7 are crucial for understanding and
its limited membership, the G-7 can still engage in effectively addressing global challenges.
fruitful collaborations with other entities to address Explanation:
global challenges. If this is true, it challenges the In this question, the assumed argument is that the
argument's claim that the G-7's limited limited representation of the G-7 hinders its
representation hinders its effectiveness in addressing effectiveness in addressing global challenges. To
global challenges. weaken this argument, we need to identify the
Option A states that the G-7 summit in Hiroshima assumption underlying it. Option A is the correct
resulted in significant commitments from member answer because it assumes that the perspectives and
countries to address global challenges. While this inputs of countries outside the G-7 are crucial for
information may highlight the positive outcomes of understanding and effectively addressing global
the summit, it does not directly address the challenges. If this assumption is false, then the
argument's claim about the G-7's limited limited representation of the G-7 may not necessarily
representation and its impact on effectiveness. hinder its effectiveness, as it implies that the G-7
countries can adequately address global challenges
without the involvement of non-G-7 countries.

[41]
Option B assumes that the G-7 countries have not challenges. Option D is the correct answer because it
been successful in addressing global challenges in aligns with the passage's suggestion that the G-20
the past due to their limited representation. While summit, with its broader representation, should
this may be a possible assumption, it is not necessary prioritize building a more comprehensive global
for the argument. The argument focuses on the consensus on major challenges.
limited representation itself as the hindrance, Option A suggests that the G-7 should immediately
regardless of past successes or failures. expand its membership to include China and India.
Option C assumes that the exclusion of certain While this could be a possible course of action, the
countries from the G-7 significantly impacts the passage does not explicitly recommend immediate
diversity of perspectives and expertise in addressing expansion or specify China and India as the only
global challenges. Although this assumption is countries to be included.
related to the argument, it is not necessary for the Option B focuses on promoting transparent financing
argument to hold true. The argument could still and debt sustainability for the developing world.
assert that the limited representation itself hinders While the passage mentions efforts in Hiroshima to
effectiveness without explicitly mentioning the recognize the G-7's role in these areas, it does not
impact on diversity. provide strong support for this course of action as
Option D assumes that the G-20, with its broader the most appropriate response to the G-7's
representation, has proven to be more effective in limitations.
addressing global challenges compared to the G-7. Option C proposes reinstating Russia's membership
While this may be true, it is not necessary for the in the G-7 to enhance its credibility. However, the
argument. The argument focuses on the limited passage mentions the expulsion of Russia due to its
representation of the G-7 and does not directly actions in Georgia and Crimea, and there is no
compare its effectiveness to the G-20. indication or suggestion in the passage that
Hence, option A is the correct answer as it captures reinstating Russia is a supported course of action.
the necessary assumption for the argument that the Option D suggests that the G-20 summit, which is
perspectives and inputs of countries outside the G-7 mentioned in the passage as the next major event,
are crucial for understanding and effectively should prioritize building a comprehensive global
addressing global challenges. consensus considering its broader representation.
This aligns with the passage's focus on the
14. Correct Option D limitations of the G-7's representation and the hope
(H & S) for a more inclusive outlook at the G-20 summit.
Answer: D) The G-20 summit should prioritize Hence, option D is the most supported course of
building a more comprehensive global consensus on action as it corresponds to the passage's emphasis on
major challenges, considering its broader the G-20 summit and the need for a broader
representation. representation to address global challenges
Explanation: In this course of action question, we effectively.
need to identify the most supported course of action
based on the information presented in the passage. 15. Correct Option A
The passage discusses the limitations and (H & S)
unrepresentative nature of the G-7, emphasizing the Answer: A) The G-7's limited membership
need for a broader perspective in addressing global undermines its ability to address global challenges

[42]
effectively and hinders the representation of diverse Option C: The G-7's focus on economic influence
viewpoints. and political stability in membership criteria neglects
Explanation: The main argument presented in the the perspectives and interests of countries from the
passage is that the G-7's limited membership has Global South. Although the passage highlights the
several detrimental effects, including its inability to lack of representation of countries from the Global
effectively address global challenges and the lack of South in the G-7, it does not specifically mention the
representation of diverse viewpoints. Let's break membership criteria based on economic influence
down the options and explain why option A and political stability. Option C introduces a new
accurately captures the main argument: element that is not explicitly discussed in the
Option A: The G-7's limited membership passage.
undermines its ability to address global challenges Option D: The G-7's smaller population and GDP
effectively and hinders the representation of diverse representation compared to its wealth representation
viewpoints. This option accurately summarizes the diminishes its influence on global economic
main argument presented in the passage. The decisions and calls for a more inclusive outlook.
passage argues that the G-7's limited membership, While the passage does mention the smaller
consisting mainly of countries with similar population and GDP representation of the G-7, it
backgrounds, renders it unrepresentative of the does not argue that this diminishes its influence on
global community. This limited representation global economic decisions. Option D adds the notion
hampers the G-7's effectiveness in tackling global of calling for a more inclusive outlook, which is not
challenges as it overlooks diverse perspectives and the primary focus of the passage.
expertise from countries that are not included in the Therefore, option A accurately captures the main
group. argument presented in the passage by highlighting
Option B: The G-7's exclusionary approach in the negative consequences of the G-7's limited
membership selection promotes a Euro-American membership on its effectiveness and representation
worldview and perpetuates polarized views of the of diverse viewpoints.
world. While the passage mentions the Euro-
American worldview of the G-7, it does not PASSAGE IV
explicitly argue that this approach perpetuates 16. Correct Option A
polarized views of the world. Option B goes beyond (H & S)
the scope of the passage and introduces a new The correct option is (A) The convergence on
concept that is not fully supported by the argument. questions of language in 20th-century philosophy.
The argument in the passage focuses on the lack of Explanation: The passage discusses the convergence
representativeness and inclusivity of the G-7 rather on questions of language as a central theme in 20th-
than the direct perpetuation of polarized views. The century philosophy. It highlights the shift towards
passage highlights the fact that the G-7's linguistic philosophy and the emphasis on the nature
membership does not include countries from the of language in philosophical inquiry during that
Global South, and it calls for a more comprehensive time.
global consensus on global challenges. The primary Option B is incorrect option. While the author does
concern expressed is the need to broaden the G-7's mention about such rivalry, this point appears to be
prism and include perspectives from countries with only a supporting information and not the main idea
different viewpoints and backgrounds. of the passage.

[43]
Option C is incorrect option. This option is very
close to Option A. The passage does talks about Option A is incorrect as the passage does provide
Jacques Derrida's role in academic philosophy, it enough information and evidence to support the
also brings other aspects onto the table. claim that analytic and continental philosophy
Option D is incorrect option. This option is factually emerged at the same time.
incorrect as there was a split and not a shift in the Option B is incorrect because the passage doesn't
20th century. overgeneralise the information.
Hence, the correct option is A. Option D is incorrect because these aspects are not
relevant to our discussion.
17. Correct Option C Hence, the correct option is C.
(H & S)
The correct option is (C) Both analytic and 19. Correct Option C
continental philosophy have limitations and (H & S)
drawbacks. Solutions
Explanation: The passage mentions that both The correct option is (C) The convergence on
analytic and continental philosophy had similar questions of language led to groundbreaking insights
obsessions and drawbacks, indicating that there were and advancements in philosophical thought.
limitations inherent to both approaches. This Explanation: The passage discusses how the focus on
suggests the assumption that both branches of language in philosophy during the 20th century had a
philosophy have their own drawbacks. significant impact. Choosing option (C) strengthens
Option (A) is incorrect because negating this option the argument by acknowledging that the emphasis on
would not affect the passage in any way whatsoever. language yielded valuable insights and
Option (B) is incorrect because this appears to be advancements in philosophical thought.
more of an inference and less of an assumption. We Option A and Option C are very close to correct
can eliminate this option. options. However, Option C gives a more detailed
Option (D) is incorrect because negating this option information in comparison to Option A. Therefore,
would not affect the passage.0020 Option A is eliminated.
Hence, the correct option is C. Option B is incorrect because this would not affect
the passage in any way. This option is out of scope
18. Correct Option C of discussion.
(H & S) Option D is incorrect this would rather weaken
The correct option is (C) The passage contention instead of strengthening it.
overgeneralizes the perspectives of analytic and Hence, the correct option is C.
continental philosophy, failing to acknowledge the
diversity within each discourse. 20. Correct Option D
Explanation: The passage describes the rivalry (H & S)
between analytic and continental philosophy and Solutions
presents their perspectives in a contrasting manner. The correct option is (D) Understanding both
However, it overlooks the fact that there is diversity analytic and continental philosophy was a rare
within each discourse, and not all philosophers achievement due to their divergent vocabularies and
within those traditions share the same perspectives. perspectives.

[44]
focus of the passage, which is the nature and
Explanation: The passage mentions that analytic and significance of hooks in music.
continental philosophy were separate discourses, D) Hooks are musical elements that interact with the
each with its own jargon and canon. It also implies listener's mind and memory - This is incorrect
that mastering both was challenging, and few because this option only captures a specific aspect
individuals had the motivation to do so. Therefore, it discussed in the passage but does not encompass the
can be inferred that understanding both analytic and broader main idea.
continental philosophy was a rare achievement due Hence, option A is correct
to their divergent vocabularies and perspectives.
Option A is incorrect because this information is in 22. Correct Option C
contradiction with the information in the passage and (H & S)
therefore cannot be inferred. C) Hooks play a significant role in making a song
Option B is incorrect because we cannot defend this memorable and appealing to listeners.
contention with 100% surety that this outcome did Explanation: The passage emphasizes the role of
happen. hooks as memorable and attention-grabbing
Option C is incorrect because this is again in moments in music. It discusses how hooks are
contradiction with the information present in the intended to stick in a person's memory and transcend
passage. personal taste. This assumption underlies the
Hence, the correct option is D. discussion of hooks in music, suggesting that they
contribute to the song's memorability and appeal to
PASSAGE V listeners.
21. Correct Option A Incorrect options:
(H & S) A) Hooks are intentionally created by musicians to
A) Hooks in music are memorable and attention- make their songs more popular - This is incorrect
grabbing moments that stick in a person's memory. because the passage does not explicitly mention that
Explanation: The passage discusses hooks in music hooks are created with the intention of making songs
as memorable and attention-grabbing moments that more popular, although hooks do contribute to a
stand out and are easily remembered by listeners. It song's appeal.
explains the concept of hooks and provides examples B) All listeners have the same reaction and
from various genres of music. This option perception of hooks in music - This is incorrect
summarizes the main idea of the passage accurately. because the passage acknowledges that hooks can be
Incorrect options: subjective and may vary in their impact on different
B) Pop music relies heavily on hooks to capture individuals.
listeners' attention - This is incorrect because the D) The concept of hooks in music originated in the
passage acknowledges that hooks are prevalent in 1980s - This is incorrect because the passage
pop music, but it also highlights hooks in other kinds mentions that the definition of hooks is based on one
of music, such as classical. put forth in the 1980s, but it does not suggest that
C) Hooks can be found in various forms of music, hooks themselves originated in that era.
from pop to classical - This is incorrect because Hence, Option C is correct.
although the passage mentions hooks in various
forms of music, this option does not capture the main

[45]
examples like "Bad Romance" by Lady Gaga and
23. Correct Option C "Shake It Off" by Taylor Swift, which are packed
(H & S) with musical moments that stand out and are easily
Solutions remembered. This option aligns with the evidence
C) Hooks have the ability to transcend personal taste provided in the passage.
and stick in a person's memory. Incorrect options:
Explanation: The passage discusses how hooks in B) Hooks in music are solely dependent on the skill
music can transcend personal taste and be easily and creativity of the musicians - This is incorrect
remembered by listeners. It mentions examples of because the passage does not discuss the sole
hooks that stand out and stick in a person's memory, dependence of hooks on the skill and creativity of
regardless of individual preferences. This supports musicians. It acknowledges that hooks can come in
the inference that hooks have the ability to transcend various forms and can be subjective.
personal taste and leave a lasting impression. C) All listeners have the same level of attention and
Incorrect options: memory capacity when it comes to hooks - This is
A) Hooks in music are solely based on the lyrics and incorrect because the passage does not make any
vocal elements - This is incorrect because the claims about the uniformity of listeners' attention
passage mentions that hooks can come in many and memory capacity when it comes to hooks. It
forms, not solely based on lyrics and vocal elements. acknowledges that hooks can have different impacts
B) All pop songs require hooks in every section to on individuals.
hold listeners' attention - This is incorrect because D) Hooks in music are designed to cater to
the passage mentions Jay Brown's suggestion that individual preferences and personal taste - This is
modern pop songs need hooks in every section, but it incorrect because the passage discusses hooks as
does not generalize this requirement to all pop songs. moments that can transcend personal taste and have
D) Hooks in classical music are primarily based on an impact regardless of individual preferences. It
complex melodies and harmonies - This is incorrect does not suggest that hooks are specifically designed
because the passage provides examples of hooks in to cater to personal taste.
classical music that are not necessarily based on Hence, option A is correct.
complex melodies and harmonies. It mentions
Beethoven's 5th Symphony and Bill Evans's 25. Correct Option C
descending piano chords as examples of hooks in (H & S)
classical music. C) Hooks are musical moments that interact with the
Hence, option C is correct. listener's attention and memory.
Explanation: This option must be true based on the
24. Correct Option A information provided in the passage. The passage
(H & S) defines hooks as memorable and attention-grabbing
A) Many successful pop songs rely heavily on hooks moments in music that stick in a person's memory. It
to capture and retain listeners' attention. also mentions that hooks are a product of the
Explanation: This option strengthens the argument interaction between the music and the listener's
presented in the passage by emphasizing the role of capacities for attention and memory. Therefore, it
hooks in capturing and retaining listeners' attention can be concluded that hooks are musical moments
in successful pop songs. The passage mentions that interact with the listener's attention and memory.

[46]
Incorrect options:
A) All listeners remember and appreciate the same 3. Correct Option B
hooks in music - This is incorrect because the (H & S)
passage acknowledges that hooks can vary in their Option B
impact on different individuals and can be ICJ was established in 1945 by the United Nations
subjective. charter and started working in April 1946. It is the
B) Hooks in music are limited to pop songs and principal judicial organ of the United Nations,
cannot be found in other genres - This is incorrect situated at the Peace Palace in The Hague
because the passage provides examples of hooks in (Netherlands). Unlike the six principal organs of the
various genres, including pop, classical, and film United Nations, it is the only one not located in New
music. It does not limit the presence of hooks to a York (USA). It settles legal disputes between States
specific genre. and gives advisory opinions in accordance with
D) Hooks in music are solely defined by catchy international law, on legal questions referred to it by
lyrics and memorable melodies - This is incorrect authorized United Nations organs and specialized
because the passage mentions that hooks can come agencies. It has 193 state parties.
in many forms, including beats, series of notes,
lyrics, or other elements in a piece of music. It does 4. Correct Option C
not restrict the definition of hooks to catchy lyrics (H & S)
and melodies only. Option C
Hence, Option C is correct. Isreal's stand was:
1. Before the vote, Israel's Ambassador to the United
Section III Nations Gilad Erdan said that the "outrageous
PASSAGE I resolution" calling for the advisory opinion of the
1. Correct Option B International Court of Justice is a "moral stain on the
(H & S) U.N. and every country that supports it.
Option B 2. No international body can rule that Jews are
In the 1967 war, Israel occupied these territories "occupiers" in their own country. Any decision made
(Gaza Strip and the West Bank) including East by a judicial body with a mandate from the morally
Jerusalem, which was subsequently annexed by bankrupt and politicized UN is completely
Israel. The war brought about a second exodus of illegitimate."
Palestinians, estimated at half a million. 3. It was stated that holding a vote on Israel on
Shabbat is another example of the United Nations'
2. Correct Option A "moral decay," which prevents Israel's position from
(H & S) being heard in a vote whose outcome is
Option A predetermined.
The U.S. and Israel voted against the resolution 4. During the United Nations General Assembly
while Brazil, Japan, Myanmar, and France were High-Level Week in September 2021, Palestinian
among those that abstained. Authority President Mahmoud Abbas stated in his
remarks that if Israel did not withdraw to the 1967
lines within a year, the Palestinians would turn to

[47]
The Hague. Hence, it was stated that today's vote flexibility; accessibility; affordability, a broad
fulfills Abbas' ultimatum acceptance by a large section of the general public;
comparatively lesser cost and growing economic
5. Correct Option D value, have great potential to make them providers of
(H & S) healthcare that the large sections of our people need.
Option D
The Permanent Representative of India to the United 8. Correct Option D
Nations is India's foremost diplomatic representative (H & S)
to the United Nations. The permanent Representative Solutions
is the head of the Permanent Mission of India to the Option D
United Nations in New York City. Currently Ruchira The Ministry of Ayush, a ministry of the
Kamboj is the Permanent Representative of India to Government of India, is responsible for developing
the United Nations. education, research and propagation of traditional
medicine systems in India. Shri Sarbananda Sonowal
PASSAGE II is the Hon'ble Cabinet Minister, Ministry of Ayush
6. Correct Option D & Ministry of Ports, Shipping and Waterways.
(H & S)
Option D 9. Correct Option B
National Commission for Indian System of Medicine (H & S)
(NCISM) and the Central Council for Research in Option B
Ayurvedic Sciences (CCRAS), the two prominent It was observed that the research potential of the
institutions under the Ministry of Ayush, have large community of Ayurveda teachers remains
launched ‗SMART‘ program. The SMART (Scope underutilized mostly. Therefore, the ‗SMART‘
for Mainstreaming Ayurveda Research in Teaching program will have a deep long term rejuvenating
Professionals) program aims to boost scientific impact on research in the field of Ayurveda and it
research through Ayurveda colleges and hospitals. It will be a great service to the nation. It aims to
was observed that the research potential of the large identify, support and promote innovative research
community of Ayurveda teachers remains ideas in healthcare research areas including
underutilized mostly. Therefore, the ‗SMART‘ Osteoarthritis, Iron Deficiency Anaemia, Chronic
program will have a deep long term rejuvenating Bronchitis, Dyslipidemia, Rheumatoid Arthritis,
impact on research in the field of Ayurveda and it Obesity, Diabetes Mellitus, Psoriasis, Generalised
will be a great service to the nation. Anxiety Disorder, Non-alcoholic fatty liver disease
(NAFLD). The program will motivate teachers for
7. Correct Option C taking up projects in designated areas of healthcare
(H & S) research and create a large database.
Option C
Ayush is Traditional & Non-Conventional Systems 10. Correct Option D
of Health Care and Healing which includes (H & S)
Ayurveda, Yoga, Naturopathy, Unani, Siddha, and Option D
Homoeopathy. The positive features of the Indian It was launched in September 2014 by the
systems of medicine namely their diversity and Department of AYUSH under the Ministry of Health

[48]
and Family Welfare, during the 12th Plan for 12. Correct Option A
implementation through States/UTs. Now, it is (H & S)
implemented by the Ministry of Ayush.The scheme Answer: A. A peace process aimed at resolving the
involves expansion of the AYUSH sector to promote Israeli-Palestinian conflict
holistic health of Indians. The Mission addresses the Explanation: The Oslo Process is a peace process
gaps in health services through supporting the efforts aimed at resolving the Israeli-Palestinian conflict. It
of State/UT Governments for providing AYUSH began after secret negotiations in Oslo, Norway,
health services/education in the country, particularly resulting in both the recognition of Israel by the PLO
in vulnerable and far-flung areas. and the recognition by Israel of the PLO as the
Components of the National AYUSH Mission: representative of the Palestinian people and as a
Obligatory Components: partner in bilateral negotiations.
1. AYUSH Services.
2. AYUSH Educational Institutions. 13. Correct Option A
3. Quality Control of ASU&H (Ayurveda, Siddha (H & S)
and Unani & Homoeopathy) Drugs. Solutions
4. Medicinal Plants. Answer: A. Adopt a balanced approach in its
Flexible Component dealings with Afghanistan
1. AYUSH Wellness Centres comprising Yoga and Explanation: To handle the situation in Afghanistan,
Naturopathy, India should adopt a balanced approach in its
2. Tele-medicine, dealings with Afghanistan, avoiding excessive
3. Innovations in AYUSH including Public Private alignment or confrontation, while expressing
Partnership, concerns about human rights, terrorism, and the
4. IEC (Information, Education and Communication) treatment of minorities.
activities,
5. Voluntary certification scheme: Project based, etc. 14. Correct Option B
(H & S)
PASSAGE III Answer: B. Oslo Summit
11. Correct Option A Explanation: Recently, Taliban representatives met
(H & S) Indian and Pakistani special envoys amongst a
Answer: A. Development initiatives that benefit the number of international diplomats, in an effort by the
Afghan people directly, such as infrastructure Norwegian Government to break the impasse in talks
development, healthcare, education, and capacity on the sidelines of a peace conference in Oslo.
building.
Explanation: Despite the Taliban takeover, India 15. Correct Option D
could continue supporting development initiatives (H & S)
that benefit the Afghan people directly, such as Answer: D. Reconstruction Opportunity Zones
infrastructure development, healthcare, education, Explanation: Reconstruction Opportunity Zones
and capacity building (ROZs) is a development initiative proposed by the
United States and supported by India to promote
economic growth and stability in Afghanistan.

[49]
PASSAGE VI 20. Correct Option B
16. Correct Option D (H & S)
(H & S) Answer: B. IT Amendment Act, 2008
Answer: D. European Union (EU) Explanation: India has some privacy provisions in
Explanation: The European Union's General Data place under the IT (Amendment) Act, 2008, although
Protection Regulation (GDPR) focuses on a these provisions are largely specific to certain
comprehensive data protection law for processing situations, such as restrictions on publishing the
personal data. The GDPR's strict norms regarding names of juveniles and rape victims in the media.
data transfers to third countries have had a profound
influence on data protection frameworks beyond the PASSAGE V
EU. 21. Correct Option A
(H & S)
17. Correct Option D Answer: A. It could lead to increased arms control
(H & S) concerns
Solutions Explanation: According to the given text, China
Answer: D. B.N. Srikrishna Committee could potentially have as many Intercontinental
Explanation: The Indian government appointed a Ballistic Missiles (ICBMs) as the US or Russia by
committee of experts for data protection under the the end of the decade. This could lead to increased
chairmanship of Justice B.N. Srikrishna in August arms control concerns.
2017, which submitted its report in July 2018 along
with a draft Data Protection Bill. 22. Correct Option A
(H & S)
18. Correct Option A Answer: A. It could lead to increased tensions
(H & S) between India and China
Answer: A. Personal Information Protection Law Explanation: According to the given text, India's
(PIPL) nuclear deterrent is placing growing emphasis on
Explanation: The Personal Information Protection longer-range weapons capable of reaching targets
Law (PIPL) came into effect in November 2021 in across China. This could lead to increased tensions
China, which gives Chinese data principals new between India and China.
rights as it seeks to prevent the misuse of personal
data. 23. Correct Option C
(H & S)
19. Correct Option B Solutions
(H & S) Answer: C. It highlights the global imbalance of
Solutions power in terms of nuclear weapons
Answer: B. Data Protection Board of India Explanation: According to the given text, Russia and
Explanation: The central government will establish the United States possess almost 90% of all nuclear
the Data Protection Board of India to adjudicate non- weapons. This highlights the global imbalance of
compliance with the provisions of the Personal Data power in terms of nuclear weapons.
Protection Bill.

[50]
24. Correct Option A interest in the craters present in the southern pole.
(H & S) They believe these cold traps may contain
Answer: A. Increased arms control concerns and mysterious fossil records of the early planetary
regional instability system.
Explanation: According to the given text, India's Hence Option D is correct.
nuclear arsenal is expanding, which could lead to
increased arms control concerns and regional 27. Correct Option A
instability. (H & S)
Ans-a) I. Mars Orbiter Mission (MOM), also called
25. Correct Option B Mangalyaan, is a spacecraft orbiting Mars since 24
(H & S) September 2014. It was launched on 5 November
Solutions 2013 by the Indian Space Research Organisation
Answer: B. 12,512 (ISRO). It is India's first interplanetary mission and
Explanation: According to the Stockholm ISRO has become the fourth space agency to reach
International Peace Research Institute (SIPRI), as of Mars, after the Soviet space program, NASA, and
January 2023, the total global inventory of warheads the European Space Agency. India is the first Asian
is estimated at 12,512. nation to reach Mars orbit, and the first nation in the
world to do so in its first attempt. It is correct.
PASSAGE VI II. ASTROSAT is the first dedicated Indian
26. Correct Option D Astronomy satellite mission not earth observatory
(H & S) launched by ISRO on 28 September 2015, which
Ans-d) Option A- Chandrayan 3 would consist of a enabled multi-wavelength observations of the
lander and a rover similar to the Chandrayaan-2 but celestial bodies and cosmic sources in X-ray and UV
would not have an orbiter like Chandrayan 2. The spectral bands simultaneously. It is not correct.
Propulsion module will carry the lander and rover III. NASA-ISRO Synthetic Aperture Radar (NISAR)
configuration till 100 km lunar orbit. is a NASA-ISRO collaboration to co-develop and
Option B- The Chandrayaan-3 lander will be deploy a dual-frequency synthetic aperture radar
equipped with a Laser Doppler Velocimeter (LDV). satellite for remote sensing not European space
LDV sensors will be useful as they will help directly agency ESA. It holds the distinction of being the first
measure the velocity from the height of about 20km. dual-band radar imaging satellite.
Option C- Chandrayaan 3 spacecraft is the 3rd lunar Hence option A
exploration expedition, outlined by the ISRO. The
first lunar is Chandrayan 1 launched in 2008 and the 28. Correct Option B
second is Chandrayan 2 which failed to reach the (H & S)
targeted area in 2019. Solutions
Option D- One key reason to target the Moon‘s Ans-b) Chandrayaan-3 rover will touch down in the
southern pole with Chandrayaan 3 is that it contains lunar south pole‘s Aitken basin. It is of particular
larger shadowed areas than the northern pole. interest as it is thought to host numerous deposits of
Scientists believe that these areas on the lunar subsurface water ice – a vital component for any
surface possibly have a permanent source of water, future sustainable lunar habitation.
not oxygen. In addition, scientists also have a keen

[51]
29. Correct Option C Option B can be inferred from this line of the
(H & S) passage- "Section 34 of the IPC makes a co-
Ans c) The CE-20 cryogenic engine, which will perpetrator, who had participated in the offence,
power the Cryogenic Upper Stage of the LVM3, a equally liable on the principle of joint liability."
flight acceptance hot test that was successfully Optiom C can be inferred from this line of the
completed. A 25-second hot test was performed at passage- "The High court partly allowed the appeal
the ISRO Propulsion Complex in Mahendragiri, filed by Gurbachan Singh on the ground that
Tamil Nadu. common intention could not be inferred from his
conduct, as he was only armed with 'lathi' and had
30. Correct Option C struck only on the feet of Teja Singh."
(H & S) Hence the correct option is D.
Ans c) K Sivan was the chairman of Isro from 2018-
2022. He also contributed to Chandrayan 2 and 3. 3. Correct Option C
Chandrayan 2 eventually failed. (H & S)
Option C.
Section IV Explanation: This can be inferred from the following
PASSAGE I given in the passage: ―The High court partly allowed
1. Correct Option C the appeal filed by Gurbachan Singh on the ground
(H & S) that common intention could not be inferred from his
Option C. conduct, as he was only armed with 'lathi' and had
Explanation: In the observation given by the Apex struck only on the feet of Teja Singh.‖
Court,I and III are mentioned in the following lines: Hence the correct option is C.
―Common intention can be formed at the spur of the
moment and during the occurrence itself.‖ 4. Correct Option B
―Common intention is necessarily a psychological (H & S)
fact and as such, direct evidence normally will not be Option B.
available.‖ Explanation: This can be inferred from the following
Hence the correct option is C. given in the passage: ―the Apex court disagreed with
this finding of the High Court and observed that the
2. Correct Option D common intention to inflict injuries and cause the
(H & S) death of Teja Singh, can be gathered from the
Option D. conduct and action of Gurbachan Singh.‖
Explanation: All the options are correct and can be Hence the correct option is B.
inferred from the passage.
Option A can be inferred from this line of the 5. Correct Option C
passage- "Common intention can be formed at the (H & S)
spur of the moment and during the occurrence itself. Option C.
Common intention is necessarily a psychological Explanation: Common Intention is different from the
fact and as such, direct evidence normally will not be same or similar intention.
available." Hence the correct option is C.

[52]
PASSAGE II Thus, the criminal liability cannot shift to Moina
6. Correct Option C since she has no legal obligation to prove anything.
(H & S) Hence the correct option is B.
Option a – Statutory rape law is violated in
circumstance where an individual engages himself in 8. Correct Option C
the sexual activity with a person under age of 18. (H & S)
The law mandates that although she willingly engage Explanation:
in sexual activity, the sex isn‘t consensual. Option a – Kusha and Deepak both did not commit
Option b – Intention to marry or any other plans are statutory rape under the law in India. Only Deepak
immaterial to statutory rape. The law mandates that committed the offence of statutory rape since Kusha
although he or she willingly engage in sexual was a minor woman incapable of giving consent.
activity, the sex isn‘t consensual. Option b – Deepak is criminally liable of statutory
Option c – Statutory rape law is violated in rape but only because Kusha was a minor woman
circumstance where an individual engages himself in and Deepak‘s age is immaterial to the case. His
the sexual activity with a person under age of 18. minority does not negate his criminal offence.
The law mandates that although he or she willingly Option c – Statutory rape law is violated in
engage in sexual activity, the sex isn‘t consensual. circumstance where an individual engages himself in
Option d – Major partner‘s consent to the act is the sexual activity with a person under age of 18.
immaterial and the aim of statutory rape laws is to Only Kusha was a victim of statutory rape and only
protect young minor females from males who might her consent is immaterial. The cause of Deepak‘s
exploit them. acts does not save him from any criminal liability.
Hence the correct option is C. Option d – The passage does not talk about the
informant. Further, it does not bar 3rd parties from
7. Correct Option B informing the police.
(H & S) Hence the correct option is C.
Explanation:
Option a – The law does not talk about any 9. Correct Option C
disclosure of age and does not place any burden on (H & S)
the minor since they are incapable to taking Explanation:
decisions for themselves. Thus, it was not Moina‘s Option (a)– the age of consent and majority are not
duty to disclose her age. dependent on the age of marriage. The age of
Option b – The only condition for statutory rape is consent and majority remains 18 even if the law
involvement of a minor woman in the intercourse. regarding the age of marriage changes. Thus,
This condition is fulfilled and no actions or although Madhav and Mira were eligible to get
intentions are relevant to disprove statutory rape. married under the new law, the consummation of
Option c – The bone of contention is not the marriage remains illegal since Mira was 16 and had
intended marriage on attaining majority but the act not attained the age of majority.
of engaging in sexual intercourse with a minor. Option (b) – the preponement of marriage of
Option d – The law does not talk about any liability marriage was legal since they were legally eligible to
shift and the same cannot be assumed. Additionally, be married under the same law. However, they could
her consent for sexual intercourse is immaterial. not consummate the marriage until Mira has attained

[53]
the age of majority. Thus, there is no relation of the
legality of preponement of marriage with the consent Hence the correct option is C.
age.
Option (c) – this option highlights the illegality of 12. Correct Option C
the consummation of marriage and does not (H & S)
formulate any relationship between the two ages. Explanation: Option A is incorrect as in the question
Option (d) – Madhav and Mira were eligible to get liability of A and B is asked. 2nd para last 5th line
married under the new marriage law though their act states that it might happen during the occurrence of
of consummation of marriage was illegal. the act as well. The intention is curated at any given
Hence the correct option is C. point in the action. Usually, an overt act would be
sufficient to show that there is common intention.
10. Correct Option C Last line also states that the law requires that the
(H & S) accused must be present on the spot during the
Explanation: occurrence of the crime and take part in its
Option (a) – It applies extra legal knowledge not commission; it is enough if he is present somewhere
provided on the passage. nearby. On this reasoning Along with A, B shall also
Option (b) – It applies extra legal knowledge not be liable for the very fact that B was waiting for A to
provided on the passage. stab and then run away with him in a jeep shows that
Option (c) – Refer to ―The aim of statutory rape laws there was a meeting of minds and thus common
is to protect young minor females from males who intention was formed. Thus option C is correct and
might exploit them and do not take responsibility by on same reasoning option B is incorrect.
giving support for the child‖. Thus, it is a protection Hence the correct option is C.
against men leaving the minor girl after
impregnating them thus, leaving the burden of 13. Correct Option D
raising the child on her. (H & S)
Option (d) – It is inconsistent with the facts and Solutions
applies extra legal knowledge. Explanation: Option D is correct, para last 5th line
Hence the correct option is C. states that it might happen during the occurrence of
the act as well. The intention is curated at any given
PASSAGE III point in the action. Usually, an overt act would be
11. Correct Option C sufficient to show that there is common intention.
(H & S) Last line also states that the law requires that the
Explanation: There is nothing in the facts that accused must be present on the spot during the
suggests that all the three accused had the chance to occurrence of the crime and take part in its
pre-arrange or a chance so that there could be a commission; it is enough if he is present somewhere
meeting of minds. Thus, it cannot be said that there nearby. On this reasoning it cannot be said that mere
was common intention. Thus, option C is correct. On joining of Chetan in the escape jeep amounts to
the same reasoning as there was no pre meeting of meeting of minds or pre-meditation. There was no
mind thus options A and B stands incorrect. As C is chance of a common intention being developed with
the correct answer with correct reasoning hence Chetan who was not aware of the stabbing in this
option D is eliminated. case. Thus option A is incorrect. On the same

[54]
reasoning the correct answer is (d). Option B is criminal act for common intention to apply. As a
incorrect as Boru was waiting right next to the place result The correct answer is (d). on the same
of murder on a jeep so that they can run away so reasoning Options A and C are incorrect. Option B is
common intention was formed on the spot during the incorrect as the reasoning is not correct. They both
occurrence of the crime. Hence option B is incorrect. have planned to get marry which shows pre-
Option C does not provide appropriate legal meditation but getting married is not a criminal act
reasoning and passage is silent on self –defence. so it stands incorrect.
Thus it stands cancelled.
Hence the correct option is D. PASSAGE IV
16. Correct Option B
14. Correct Option D (H & S)
(H & S) The correct option is b- No, he did not have the
Option D. intention to hit the old lady with the ball.
Explanation: Since both A and B were adults (as To hold a person liable for trespass, it is important to
they loved each other for 25 years now), they had the establish that he had the intention of doing that act.
right to run away and marry each other. Thus, no In this case, the intention is not there and thus
criminal act had been committed in the instant Shyam cannot be helped liable for trespass.
case.The correct answer is option (d).Options A and Option d is not consistent with the facts of the
C are incorrect as 4th line of the 2nd para clearly situation and options a and c are not complying with
states that for common intention one of the basic the information in the passage. Hence, options a,c
most requirements is that there must be some and d are incorrect.
criminal act. In present situation there is no criminal Hence the correct option is B.
act thus stands incorrect. Option B is incorrect as the
reasoning is not correct. They both have plannedto 17. Correct Option D
get marry which shows pre-meditationbut getting (H & S)
married is not a criminal act so it stands incorrect. Option a is a case of contributory negligence and
option b is a case of self-defence. Both of these can
15. Correct OptionD be used as defences as inferred from the passage.
(H & S) Option c is incorrect as a necessity can be taken up
Option D. as a defence as inferred from the passage.
Explanation: 4th line of the 2nd para of the passage Hence the correct option is D.
clearly states that one of the basic most requirements
is that there must be some criminal act. Such a 18. Correct Option C
criminal act must be done by ―several persons.‖ Such (H & S)
a criminal act however would not include acts which The correct option is C- No, as Neha acted out of
are merely invalid in the law. An example would be necessity.
a minor man marrying a minor woman for the Even though Neha had the intention of pushing
marriage is merely void and not a crime. The Ayush, she did the act out of necessity, which is a
principle pertaining to the Marriage only renders a defence for trespass.
marriage void. It does not state that marrying in such
manner is an offence. Thus, there is not a constituted

[55]
Thus, option C is correct and option A is incorrect. The correct option is D- Both a and b.
Options b and d cannot be inferred from the passage Both the statements- a and b can be inferred from the
and are thus incorrect. passage whereas option c cannot be inferred from the
Hence the correct option is C. passage.

19. Correct Option C 22. Correct Option C


(H & S) (H & S)
The correct option is C- Both 1 and 2. The correct option is C- No, the reason is unjustified
Both the statements can be inferred from the passage because the oversights committee has to purchase
as trespass to person is the causing of apprehension and distribute the CCTVs.
of unreasonable interference with one‘s person and Thus, options a, b and d are incorrect.
body as well as a third person and includes usage of
force causing damage and 23. Correct Option B
impairment in the body. The trespasser, with an (H & S)
ulterior intention, transgresses the right of another The correct option is b- The state government under
and makes an alteration in it with the objective to Section 30 of the Human Rights Act, has the power
cause wrongful loss or wrongful gain as the case to establish district human rights courts.
may be. Option b has been observed in the passage whereas it
has been stated that the Supreme Court and the High
20. Correct Option A Court do not have the authority to establish district
(H & S) human rights courts. Tus, options a, c and d are
Solutions incorrect.
The correct option is A- Ishaan's act of throwing
water at Hardik was intentional and thus, he can be 24. Correct Option B
held liable for trespass002E (H & S)
A is correct as intention is the main element for The correct option is B- According to the guidelines
establishing the tort of trespass. If there is an by the Supreme Court, CCTV footage can be stored
intention behind committing a trespass then it is in any recording system wherein data is preserved
actionable per se and the plaintiff need not prove that forever.
any damage has been caused to him due to the act of This statement is incorrect as according to the
the accused. Thus, option B is incorrect. Option C is Supreme Court, the recording has to be preserved for
incorrect as no trespass was done on the part of a period of 18 months and not forever.
Hardik and option D cannot be inferred from the All other statements can be inferred from the
passage. passage. Thus, options a, c and d are incorrect.

PASSAGE V 25. Correct Option C


21. Correct Option D (H & S)
(H & S) The correct option is C- As per the guidelines of the
Supreme Court, CCTVs are to be installed in places
where the police conduct interrogation and the police
station is such a place.

[56]
treasure, it fulfils all the essential of theft. So, A
Option A is incorrect as CCTVs have to be installed liable for theft
in places where interrogation is conducted. Options b
and d are incorrect as it is necessary to install Section V
CCTVs in police stations. PASSAGE I
1. Correct Option B
PASSAGE VI (H & S)
26. Correct Option B English Hindi Science History Art
(H & S) Aman 90 120 135 75 80
Hence, the correct option is (B)The moment Mr. X
Baman 85 115 125 80 70
moved the bracelet from it‘s original position with
Chaman 40 60 80 120 130
the intention to steal it , he has committed theft ,
regardless of consent of Mr. Y or the fact that the
The average marks obtained by Aman in all subjects = 90
necklace has not been moved out of Mr.Y‘s house
Total marks of Aman  5  100  500

27. Correct Option C Marks obtained by Aman in Art = 500 - 420 =80

(H & S) Total marks obtained by Baman = 5  95  475

Hence, the correct option is (C)The farm house has Marks obtained by Baman in English

high walls, so taking something without consent be it = 475 - 115 – 125 – 80 - 70 = 85

be natural thing severed from the tree without Total marks obtained by Chaman = 5  80  400

consent , it amounts to theft, Marks obtained by Chaman in Science =


 3  60  3  100  400  480  400  80

28. Correct Option C 80: 85 =16: 17

(H & S)
Hence, the correct option is (C) Mrs. V is not liable 2. Correct Option C
for theft because mens rea or dishonest intention is (H & S)
one of the necessary ingredient to prove theft. Her English Hindi Science History Art
intentions were bonafide in nature ,her act doesn‘t Aman 90 120 135 75 80
amount to theft.
Baman 85 115 125 80 70

Chaman 40 60 80 120 130


29. Correct Option B
(H & S)
Hence, the correct option is (B) Mrs. Lata The average marks obtained by Aman in all subjects = 90

dishonestly taken something out of the possession of Total marks of Aman  5  100  500

someone else, the jewels being the authority in this Marks obtained by Aman in Art = 500 - 420 =80

case, would amount to act of theft. Total marks obtained by Baman = 5  95  475
Marks obtained by Baman in English

30. Correct Option D = 475 - 115 – 125 – 80 - 70 = 85

(H & S) Total marks obtained by Chaman = 5  80  400


Hence, the correct option is (D) As A dishonestly Marks obtained by Chaman in Science =
moves the bullock in a certain direction to take the  3  60  3  100  400  480  400  80

[57]
Marks scored by Chaman in history = 75 + 25 =120 Marks obtained by Chaman in Science =
Percentage of marks scored by Chaman in history  3  60  3  100  400  480  400  80

120 Marks obtained by Chaman in Englsih =


= 100  80% 3  60  80  60  40
150
Required ratio = 40: 90 =4: 9
3. Correct Option D
(H & S) 5. Correct Option A
English Hindi Science History Art (H & S)

Aman 90 120 135 75 80 English Hindi Science History Art

Baman 85 115 125 80 70 Aman 90 120 135 75 80

Chaman 40 60 80 120 130 Baman 85 115 125 80 70

Chaman 40 60 80 120 130


The average marks obtained by Aman in all subjects = 90
Total marks of Aman  5  100  500 The average marks obtained by Aman in all subjects = 90
Marks obtained by Aman in Art = 500 - 420 =80 Total marks of Aman  5  100  500
Total marks obtained by Baman = 5  95  475 Marks obtained by Aman in Art = 500 - 420 =80
Marks obtained by Baman in English Total marks obtained by Baman = 5  95  475
= 475 - 115 – 125 – 80 - 70 = 85 Marks obtained by Baman in English
Total marks obtained by Chaman = 5  80  400 = 475 - 115 – 125 – 80 - 70 = 85
Marks obtained by Chaman in Science = Total marks obtained by Chaman = 5  80  400
 3  60  3  100  400  480  400  80 Marks obtained by Chaman in Science =
Marks obtained by Chaman in Art =  3  60  3  100  400  480  400  80
3  110  80  120  130
135  125  80 340
Required average =    113.33
Required ratio =115: 130 =23: 26 3 3
PASSAGE II

4. Correct Option D 6. Correct Option B

(H & S) (H & S)

English Hindi Science History Art Selling price of article C = Marked price =1640
Ratio between selling price and cost price of article
Aman 90 120 135 75 80
C= 1.5 = 15/10= 3:2
Baman 85 115 125 80 70
2
Cost price of article C =  1640  1093.33 =1093
Chaman 40 60 80 120 130 3
approx.

The average marks obtained by Aman in all subjects = 90 Profit on C = 1640 -1093 = 547

Total marks of Aman  5  100  500 Selling price of article D = Marked price =2450
Marks obtained by Aman in Art = 500 - 420 =80 Ratio between selling price and cost price of article
Total marks obtained by Baman = 5  95  475 D= 1.8 =18/10=9:5

Marks obtained by Baman in English 5


Cost price of article C =  2450  1361 approx.
= 475 - 115 – 125 – 80 - 70 = 85 9

Total marks obtained by Chaman = 5  80  400 Profit on D = 2450 – 1361 =1089

[58]
Required difference =1089 -547 =542 Discount on C =1640 – 1230 = 410
The cost price of article E =0.20  2450 =490
7. Correct Option A The ratio between selling price and cost price of
(H & S) article E=1.6 =16/10=8:5
Selling price of article E = 2750 -250 =2500 8
The selling price of article E =  490  784
Ratio between selling price and cost price of article 5
E =1.6 =16/10 =8: 5 Discount on E =2750 -784 =1966
5 Total discount on article C and E together
Cost price of article E =  2500  1562.5
8 =1230 + 1966 = 3196
The ratio between cost price of article A and E = 3: 5 Total marked price of article C and E together
=1640 + 2750 =4390
3
Cost price of article A = 1562.5  937.5
5 3196
Required discount % = 100  72.8%
4390
Ratio between selling price and cost price of article
A =0.8= 8/10= 4:5

4 10. Correct Option C


Selling price of article A =  937.5  750
5 (H & S)
8. Correct Option D Selling price of article D = 0.8  2450 =1960

(H & S) Ratio between selling price and cost price of article


D =1.8 =18/10 =9: 5
Selling price of article D =0.8 x 2450 =1960
3 5
Selling price of article B =  1960  2940 Cost price of article D =  1960
2 9
Ratio between selling price of article B and cost Selling price of article E = 0.90  2750 =2475
price =1.2 =6: 5 Ratio between selling price and cost price of article
2940 E =1.6 =16/10 = 8:5
Cost price of article B =  5  2450
6 5
Cost price of article E =  2475
8
9. Correct Option C 5 5
Required ratio = 1960 :  2475 =3136: 4455
(H & S) 9 8
The cost price of article C =0.5  1640 =820
The ratio between selling price and cost price of
article C=1.5= 15: 10 =3: 2

3
Selling price of article C =  820  1230
2

For more questions, kindly visit the library section: Link for app: https://links.physicswallah.live/vyJw

For more questions, kindly visit the library section: Link for web: https://physicswallah.live/tabs/tabs/library-
tab
PW Mobile APP: https://physicswala.page.link/?type=contact-us&data=open
For PW Website: https://www.physicswallah.live/contact-us

[59]

You might also like